Ethics in Medicine

¡Supera tus tareas y exámenes ahora con Quizwiz!

when the discussion occurs in the context of delivery room management of extreme prematurity, usually parents wishes (whatever they may be) are followed at _______ weeks gestation

23 and for the most part 24 weeks Whereas at 22 weeks most neonatologists are reluctant to aggressively resuscitate and at 25 weeks reluctant to withhold such care.

how old is the earth

4.5 billion years

fetus viability

<22 Completed Weeks (154 days)- Fetus is not viable 22 Completed Weeks (154 to 160 days)- Survival very infrequent. 23 to 24 Completed Weeks (161 to 174 days)- 10-50% 25 to 26 Completed Weeks (175 to 188 days)- 50-80%

I'm not sure how I feel about "using" vulnerable patients as teaching patients. Are we taking unfair advantage of people?

A necessary part of learning to be a physician, "practicing" on people sometimes feels uncomfortable. You can keep a few things in mind to minimize the discomfort you might feel. First, as with all your future patients, treat them with respect and ask permission before doing any observations, tests, or procedures. Second, remember that it is a privilege to learn medicine. When appropriate, convey your gratitude to the patients, acknowledging the crucial role they play in your education. Listen to your instincts as well. Sometimes it may not be appropriate to do an unnecessary duplicate examination or, for example, try more than three times to start an IV line in a patient. If the patient is uncomfortable with your presence, you must respect that and ask a more senior person on your team to complete the procedure or the exam. Unfortunately, you may notice a difference in how some housestaff or attending physicians treat patients from different socioeconomic classes. It is your responsibility to attend to these patients needs with respect and compassion. The homeless man in the ER could be very lucky to have you be the one to stitch his lacerations if you are the one who will be gentle and kind. Sometimes you can put a patient at ease if you convey that you are the member of the team with the most time and attention at the moment.

Can a patient demand that I provide them with a form of treatment that I am uncomfortable providing?

A physician is not morally obligated to provide treatment modalities that they do not believe offer a benefit to the patient or which may harm the patient. Physicians should also not offer treatments that they do not feel competent to provide or prescribe. However, it is important to take the patient's request seriously, consider accommodating requests that will not harm the patient or others, and attempt to formulate a plan that would be acceptable to both the physician and patient.

Proponent vs opponent in neonatal care

A proponent: Neonatal intensive care is responsible for the survival of a significant number of infants who formerly would not have survived. This increased survival has been accomplished with an acceptable level of burden and without substantially increasing the population of handicapped children. An opponent: Neonatal intensive is a good example of medicine out of control. There is inappropriate use of technology by health care professional who are out of touch with patients and their families. The benefits of increases survival of high risk infants are outweighed by the associated burdens.

Is a psychiatry consult required to determine decision making capacity?

A psychiatry consult is not required, but can be helpful in some cases. Psychiatrists are trained in interviewing people about very personal, sensitive issues, and thus can be helpful when patients are facing difficult choices with fears or concerns that are difficult to talk about. Similarly, if decision making capacity is clouded by mental illness, a psychiatrist's skill at diagnosis and potential treatment of such disorders can be helpful.

Provincial Safeguards

A regulated health professional must witness an eligibility assessment conducted via the Telehealth videoconferencing system.* (A Telehealth assessment would be arranged by the doctor or nurse practitioner.) If one or both doctors or nurse practitioners are concerned about an individual's capability to provide informed consent, they will request a capability assessment from a third doctor or specialist. The pharmacist must dispense the drugs directly to the prescribing doctor or nurse practitioner and the prescribing doctor or nurse practitioner must return any unused drugs to the pharmacy.* The doctor or nurse practitioner must be present with the person during the self-administration or administration of medical assistance in dying and remain with the person until death is confirmed. This may not be delegated to another person or professional. * These safeguards have been temporarily eased during the COVID-19 public health emergency in BC to ensure that people continue to have access to medical assistance in dying.

Do I have to do whatever I am told by the attending physician, even if I disagree with their plans?

An attending physician, for example, may be held both morally and legally liable for the actions of students or residents, whether or not she approved of those actions. Ethically, teachers have obligations to observe and control the actions of junior members of the medical team, both to prevent harm to patients from inexperienced care-givers, and to educate students in appropriate care. Students and residents, conversely, have obligations to their patients and to their teachers, to not act recklessly or without the knowledge and approval of supervisors. Whenever a student or resident disagrees with an attending physician's plans, he should seek input from the attending, both about the reasoning to pursue the attending's plan, and about the reasoning for rejecting her own. A respectful exchange of views may provide both parties with new information, and certainly serves to further education.

When should I refer to a patient's advance directive?

Appeals to living wills and surrogate decision makers are ethically and legally inappropriate when individuals remain competent to guide their own care. The assessment of decisional incapacity may be difficult at times, is thought to be a responsibility of most physicians, but sometimes may require a psychiatric evaluation. Some directives are written to apply only in particular clinical situations, such as when the patient has a "terminal" condition or an "incurable" illness. These ambiguous terms mean that directives must be interpreted by caregivers. Some more recent types of instructive directives have attempted to overcome this ambiguity by addressing specific interventions (e.g. blood transfusions or CPR) that are to be prohibited in specified clinical contexts.

The major ethical argument for the use of instructive directives, such as a living will

Appeals to the ethical principle of autonomy. Even after a patient loses the capacity to be autonomous, we can continue to respect autonomy by abiding by the patient's prior expressed wishes. In this way, patients can continue to participate (indirectly) in their medical care decisions even if they become decisionally incapacitated, - Instructive directives may extend individual autonomy and help ensure that future care is consistent with previous desires. The living will was created initially to help prevent unwanted, invasive medical care at the end of life. However, the living will also attempts to ensure that patients receive the treatment they want, which may be invasive, end-of-life care. Most importantly, the living will tries to promote patient-centered care. When a patient becomes incapacitated someone else will be required to make health care decisions. By designating a health care agent through a DPAHC, the patient's choice of a surrogate decision maker supersedes that of the state. A legal surrogate is particularly valuable for persons in non-traditional relationships or without close family. The health care agent need not be a relative of the patient, though this person should have close knowledge of the patient's wishes and views.

Do physicians have an ethical duty to disclose information about medical mistakes to their patients?

Physicians have an obligation to be truthful with their patients. That duty includes situations in which a patient suffers serious consequences because of a physician's mistake or erroneous judgment. The fiduciary nature of the relationship between a physician and patient requires that a physician deal honestly with his patient and act in her best interest.

Breaking bad news: For a patient on a medicine service whose biopsy just showed lung cancer, the agenda might be:

a) disclose diagnosis of lung cancer b) discuss the process of workup and formulation of treatment options "We will have the cancer doctors see you this afternoon to see whether other tests would be helpful to outline your treatment options"

Why do some object from "physician-assisted suicide" (PAS).

because of the associations between suicide and mental illness. They argue that, unlike the patients with impaired judgment who request suicide, terminally ill patients who request medication under the Act have the capacity to make a rational, autonomous decision to end their lives.

T/F you can ask for MAiD in an advance directive

hat the law in Canada and Quebec do not allow you ask for MAiD in an advance directive

Why have Supreme Courts have specifically thrown out the "captain of the ship" doctrine

he diversity of medical practice and the different forms of training and certifications required for specialty practice testify that different professionals have different expertise and therefore diverse levels of responsibility for individual acts in patient care. In this aspect the law is fair: the greater the authority and expertise asserted in a given act, the greater an individual's legal responsibility becomes.

"integrative medicine"

refers to an approach to patient care that fully utilizes both conventional and alternative methodologies.

Several approaches can be used to facilitate open communication with a patient. Physicians should:

sit down attend to patient comfort establish eye contact listen without interrupting show attention with nonverbal cues, such as nodding allow silences while patients search for words acknowledge and legitimize feelings explain and reassure during examinations ask explicitly if there are other areas of concern

Having a "grievous and irremediable medical condition" means:

you have a serious and incurable illness, disease or disability** you are in an advanced state of irreversible decline in capability your illness, disease or disability or that state of decline causes you enduring physical or psychological suffering that is intolerable to you and that cannot be relieved under conditions that you consider acceptable

Exception to Requirement to Provide Final Consent Medical assistance in dying may be provided to a person who has lost the ability to provide final consent if all of the following conditions are met:

The person met all other required eligibility criteria and safeguards The person was informed of the risk of losing the capacity to consent by the doctor or nurse practitioner and the person, before losing the capacity to consent, entered into a written agreement with the doctor or nurse practitioner that specifies a date on which medical assistance in dying will be provided In the written arrangement, the person consented to the administration by the doctor or nurse practitioner of a substance to cause their death on or before the date specified, if they lost their capacity to consent to receiving medical assistance in dying prior to that day The person has lost the capacity to consent to medical assistance in dying Prior to administration, the person does not demonstrate refusal or resistance, by words, sounds or gestures, to having medical assistance in dying provided The substance is provided to the person in accordance with the terms of the agreement.

Does depression or other history of mental illness mean a patient has impaired decision making capacity?

Patients with active mental illness including depression should have their decision making capacity evaluated carefully. They should not be presumed to be unable to make treatment decision. In several studies, patients voiced similar preferences for life-sustaining treatments when depressed as they did after treatment of their depression. Depression and other mental disorders should prompt careful evaluation, which may often be helped by psychiatry consultation.

What if I change my mind after requesting medical assistance in dying?

People requesting medical assistance in dying can change their mind and withdraw their request at any time and in any manner. For people requesting medical assistance in dying, whose death is not reasonably foreseeable, a mandatory reflection period of at least 90 clear days must pass between the date of first assessment by a doctor or nurse practitioner (day 0) and the day medical assistance in dying is provided (day 91 or later), unless the person's assessing doctors and/or nurse practitioners are both of the opinion that the person's loss of capacity is imminent and agree to shorten the reflection period. Immediately before the doctor or nurse practitioner provides medical assistance in dying - whether in the form of administering or providing the person with prescribed drugs for self-administration - they must confirm with the person that they are still sure that this is what they want and the person must provide consent to proceed (unless the person has entered into a written agreement in advance - see "Safeguards - Natural Death Is Reasonably Foreseeable" above).

What can a physician do with a particularly frustrating patient?

Physicians will sometimes encounter a patient whose needs, or demands, strain the therapeutic alliance. Many times, an honest discussion with the patient about the boundaries of the relationship will resolve such misunderstandings. The physician can initiate a discussion by saying, "I see that you have a long list of health concerns. Unfortunately, our appointment today is only for fifteen minutes. Let's discuss your most urgent problem today and reschedule you for a longer appointment. That way, we can be sure to address everything on your list." Or, "I know that it has been hard to schedule this appointment with me, but using abusive language with the staff is not acceptable. What do you think we could do to meet everybody's needs?" There may be occasions when no agreeable compromise can be reached between the physician and the patient. And yet, physicians may not abandon patients. When the physician-patient relationship must be severed, the physician is obliged to provide the patient with resources to locate ongoing medical care.

T/F Because arrests in the OR are often due to hemorrhage or medication effects, rather than the patient's underlying disease, physicians may feel that their actions "caused" the arrest, and they are ethically obliged to resuscitate the patient, even if the patient has clearly expressed wishes to the contrary.

But competent patients, or their appropriate surrogates, have the right to refuse medical procedures and care, even if the care is to counteract the effects of previous medical intervention.

Why do we agree to do surgery on patients with DNR orders?

Requiring such a patient to suspend their DNR orders to be a candidate for surgery uses their discomfort, pain, and desire to benefit from surgery to coerce them into accepting medical care (CPR) they do not want. Patient refusal of some medical therapy, such as CPR, does not ethically justify physicians denying them other medical therapy, such as surgery, that might benefit them.

When does a fetus or a newborn become a person?

Some ethicists purport that viability entitles the fetus to "moral personhood". Viability is the physical capacity for life independent of maternal corporeal support. They argue that newborns and fetuses participate in the social matrix, and that this social role develops over time, beginning prior to birth. Others note that it is impossible to treat fetuses as persons without treating pregnant women as if they were less than persons. The birth of the fetus results in a distinct patient towards whom medical therapy can be individually directed. As such, many believe that the moral status of a developmentally younger newborn supersedes that of an older viable fetus.

What are some examples of a decision that places a child a significant risk of serious harm?

Childhood vaccination provides an example of the kinds of factors that must be weighed in making this determination. While most physicians believe it is in a child's best interest to receive the routine childhood vaccinations and therefore recommend them to parents, they do not generally legally challenge parents who choose not to vaccinate their children. This is because in a well-vaccinated community the risk of contracting the vaccine-preventable illness and suffering harmful consequences from the infection are quite small. However, this calculation might shift if a clinician is faced with an unvaccinated child who has suffered a puncture would from a dirty nail. In the latter case, the risk of tetanus (a serious and almost always fatal disease if not prevented) has become significant, and the provider would be justified in seeking the power of the State (through a court order or involvement of child protective services) to assure that the child receives the vaccination and treatment necessary to prevent tetanus in a high risk situation.

Can a parent or guardian provide consent for a child in grade 9?

Consent forms and immunization information such as HealthLinkBC Files for immunizations given in school will be sent home. Parents or guardians and their children are encouraged to review the information, discuss it, and make a decision about immunization together. This can be used as an opportunity for adolescents to start making decisions about their own health. Adolescents will have the opportunity to make their own decision to be immunized whether or not they have a consent form signed by a parent or guardian.

What goals should I have in mind when working towards a decent death for my patient?

Control of pain and other physical symptoms. The physical aspects of care are a prerequisite for everything that follows. Involvement of people important to the patient. Death is not usually an individual experience; it occurs within a social context of family, significant others, friends, and caregivers. A degree of acceptance by the patient. Acceptance doesn't mean that the patient likes what is going on, and it doesn't mean that a patient has no hopes--it just means that he can be realistic about the situation. A medical understanding of the patient's disease. Most patients, families, and caregivers come to physicians in order to learn something about what is happening medically, and it is important to recognize their need for information. A process of care that guides patient understanding and decision making. One great physician does not equal great care--it takes a coordinated system of providers.

What is the accepted legal and ethical basis for decision-making regarding the nature of medical care in the newborn infant?

The child's best interest is legally and ethically primary and should be weighed over the family's well-being or societal concerns (using the Best Interest Standard). However, there is considerable discussion about the utility of applying this standard to neonates based on their present or future best interest. Also, much discussion is placed on the need to include the family's interests when making life and death decisions regarding severely compromised infants. In the interest of justice, societal concerns about excessive cost for aggressive care should be addressed at the policy level, rather than on an individual or case-by-case basis. Recommendations for the care of extremely premature infants should acknowledge that well-informed, competent parents are the best surrogate decision-makers for their babies.

When can confidentiality be breached?

Exception 1: Concern for the safety of other specific persons Access to medical information and records by third parties is legally restricted. Yet, at the same time, clinicians have a duty to protect identifiable individuals from any serious, credible threat of harm if they have information that could prevent the harm. The determining factor is whether there is good reason to believe specific individuals (or groups) are placed in serious danger depending on the medical information at hand. -a duty to warn third parties of imminent threats trumps a duty to protect patient confidentiality, however, it is usually difficult for a therapist or health care provider to accurately ascertain the seriousness and imminence of a threat. Exception 2: Legal requirements to report certain conditions or circumstances State law requires the report of certain communicable/infectious diseases to the public health authorities. In these cases, the duty to protect public health outweighs the duty to maintain a patient's confidence. From a legal perspective, the State has an interest in protecting public health that outweighs individual liberties in certain cases.

What happens when medical therapy is indicated for one patient, yet contraindicated for the other?

Fetal care becomes problematic when what is required to benefit one member of the dyad will cause an unacceptable harm to the other. When a fetal condition poses no health threat to the mother, caring for the fetal patient will always carry some degree of risk to the mother, without direct therapeutic benefit for her. The ethical principles of beneficence ("be of benefit") and nonmaleficence ("do no harm") can come into conflict. Because the patients are biologically linked, both, or neither, must be treated alike. It would be unethical to recommend fetal therapy as if it were medically indicated for both patients. Still, given a recommendation for fetal therapy, pregnant women, in most cases, will consent to treatment which promotes fetal health. When pregnant women refuse therapy, physicians must remember that the ethical injunction against harming one patient in order to benefit another is virtually absolute.

I just saw another caregiver tell something to my patient in a really insensitive way. What should I do?

First, examine what happened and ask yourself why the encounter went badly. If you see the patient later, you might consider acknowledging it to the patient in a way that doesn't slander the insensitive caregiver "I thought you looked upset when we were talking earlier and I just thought I should follow up on that--was something bothering you?"

What if my doctor or nurse practitioner won't perform medical assistance in dying?

For a variety of reasons, not all doctors or nurse practitioners will provide medical assistance in dying and no one will be forced to do so. For some, providing medical assistance in dying may conflict with their personal beliefs. Even if a doctor or nurse practitioner does not provide medical assistance in dying as a matter of conscience, a person can still expect to be treated with respect and be provided with information on how to access this service. This means health-care providers must not discriminate against people who make this request and must provide an effective transfer of care if they choose not to offer that care themselves.

Why long-term reform is needed

For child and family services, the goal is to design, test and implement an evidence-informed funding approach to ensure First Nations children, youth and families have access to culturally-based and substantively equal public services that meet their needs and community circumstances. The new evidence-informed funding approach will provide First Nations and First Nations-authorized service providers non-discriminatory, stable, predictable funding to deliver the services and supports to promote family wellness and address the factors linked to the dramatic over-representation of First Nations children in care. Examples of these services include culturally appropriate interventions and supports for children and young people with high needs through services such as counselling, respite and youth workers, and supports for parents experiencing multi-generational trauma and/or addictions such as family-based treatment, cultural supports and family reunification. For Jordan's Principle, the longer term goal is to design, test and implement a long-term approach to ensure there is no discrimination in the provision of health, social services and education supports, services and products for First Nations children and youth. Substantive equality and culturally appropriate interventions that respond to the unique needs of First Nations children and youth must be preserved.

Is an intervention more likely to be futile if a patient is elderly?

Futility has no necessary correlation with a patient's age. What determines whether a treatment is futile is whether or not the treatment benefits the patient. In cases where evidence clearly shows that older patients have poorer outcomes than younger patients, age may be a reliable indicator of patient benefit, but it is benefit, not age, that supports a judgment of medical futility. For patients of all ages, health care professionals should advocate for medically beneficial care, and refrain from treatments that do not help the patient.

What is the difference between futility and rationing?

Futility refers to the benefit of a particular intervention for a particular patient. With futility, the central question is not, "How much money does this treatment cost?" or, "Who else might benefit from it?" but instead, "Does the intervention have any reasonable prospect of helping this patient?"

Who decides when a particular treatment is futile?

Generally the term medical futility applies when, based on medical data and professional experience, a treating health care provider determines that an intervention is no longer beneficial Because health professionals may reasonably disagree about when an intervention is futile, all members of the health care team would ideally reach consensus. While physicians have the ethical authority to withhold or withdraw medically futile interventions, communicating with professional colleagues involved in a patient's care, and with patients and family, greatly improves the experience and outcome for all.

opening a discussion on spirituality with a patient or taking a spiritual history, HOPE acronym

H: Sources of hope, meaning, comfort, strength, peace, love and connection O: Organized religion P: Personal spirituality and practices E: Effects on medical care and end-of-life issues How this could look in real life: "many patients have religious or spiritual beliefs that affect their choices regarding medical care, and ask, "I'm wondering, (H) Where do you find comfort or hope in this time of illness? When things are tough, what keeps you going? (O) Does organized religion have a place in your life, or in your family's life? (P) Are there spiritual practices or beliefs that are important to you personally? (E) Are there ways that your personal beliefs affect your health care choices or might provide guidance as we discuss decisions about your care near the end of your life?"

The following questions guide a holistic best interests of the child approach when assessing a request to the implementation of Jordan's Principle:

Has consideration been given to the whole child and their needs, including their physical, emotional, and spiritual well-being in the context of their right to grow up as a member of their cultural group? Has consideration been given to respect and protect the rights of the child as a First Nations person? Will the action preserve a sense of identity, belonging, acceptance, and connection of the child to their community? Do you understand the child's circumstances and the potential impact on their well-being and development? Has consideration been given to protect the safety and integrity of the child's care within their family and community? Have the child's rights relating to education, health, and safety been upheld? Have the child's views been considered and given due weight in accordance with their age and maturity? Report a problem on this page Share this page

What are the ethical obligations of physicians when a health care provider judges an intervention is futile?

The goal of medicine is to help the sick. Physicians have no obligation to offer treatments that do not benefit patients. Futile interventions may increase a patient's pain and discomfort in the final days and weeks of life; give patients and family false hope; delay palliative and comfort care; and expend finite medical resources. Although the ethical requirement to respect patient autonomy entitles a patient to choose from among medically acceptable treatment options (or to reject all options), it does not entitle patients to receive whatever treatments they ask for. Instead, the obligations of physicians are limited to offering treatments that are consistent with professional standards of care and that confer benefit to the patient.

informed consent includes a discussion of the following elements:

The nature of the decision/procedure Reasonable alternatives to the proposed intervention The relevant risks, benefits, and uncertainties related to each alternative Assessment of patient understanding The acceptance of the intervention by the patient

How should I advise a patient if he believes that some family members will disagree with his wishes?

The patient should be informed that the best way to prevent disagreements is to communicate with everyone ahead of time to let them know who has been chosen as a spokesperson and what kind of approach to health care he wants.

What you need to understand to care for the dying

The patient's story - including how that person has viewed her life, the other persons important to her, and how she could bring her life to a close in a way that would be true to herself. The body - which covers the biomedical understanding of disease, and what limits and possibilities exist for that person. The medical care system available for this particular patient - knowing how you can make the system work for the patient, as well as the relevant law and ethics. understand yourself - because you, as a physician, can be an instrument of healing, or an instrument that does damage.

When can child welfare authorities become involved in regards to parents refusing treatment?

If a child or their parent refuses health care that two doctors say is necessary to preserve the child's life or health, the child welfare authorities can ask a court to overrule the refusal.

Is palliative sedation legal?

If a doctor gives deep and continuous sedation and continues IV fluids and artificial nutrition, this is clearly legal. However, if a doctor gives deep and continuous sedation and stops IV fluids and artificial nutrition, this may or may not be legal. That will depend on how far away from death the patient is. Deep and continuous sedation without IV fluids and artificial nutrition is clearly legal when the patient is very close to death. "Very close to death" means death is expected within a couple of days. Any withholding or withdrawal of IV fluids and artificial nutrition would in no way cause the patient's death. Deep and continuous sedation without IV fluids and artificial nutrition is likely legal when the patient is close to death. "Close to death" means death is expected within a couple of weeks. Any withholding or withdrawal of IV fluids and artificial nutrition would be unlikely to cause the patient's death. Deep and continuous sedation without IV fluids and artificial nutrition may be illegal when death is not expected for many months. Any withholding or withdrawal of IV fluids and artificial nutrition would cause the patient's death.

If CPR is deemed "futile," should a DNAR order be written?

If health care providers unanimously agree that CPR would be medically futile, clinicians are not obligated to perform it. Nevertheless, the patient and/or their family still have a role in the decision about a Do Not Attempt Resuscitation (DNAR) order. As described earlier, involving the patient or surrogate decision maker is essential to demonstrate respect for all people to take part in important life decisions. In many cases, patients or surrogate decision makers will agree to forgo attempting CPR following a transparent and honest discussion regarding the clinical situation and the limitations of medicine. Under these circumstances, DNAR orders can be written.

Exceptions to full informed consent are:

If the patient does not have decision-making capacity, such as a person with dementia, in which case a proxy, or surrogate decision-maker, must be found. A lack of decision-making capacity with inadequate time to find an appropriate proxy without harming the patient, such as a life-threatening emergency where the patient is not conscious When the patient has waived consent. When a competent patient designates a trusted loved-one to make treatment decisions for him or her. In some cultures, family members make treatment decisions on behalf of their loved-ones. Provided the patient consents to this arrangement and is assured that any questions about his/her medical care will be answered, the physician may seek consent from a family member in lieu of the patient.

When is it justifiable to discontinue life-sustaining treatments?

If the patient has the ability to make decisions, fully understands the consequences of their decision, and states they no longer want a treatment, it is justifiable to withdraw the treatment.Treatment withdrawal is also justifiable if the treatment no longer offers benefit to the patient.

What if CPR is not futile, but the patient wants a DNAR order?

If the patient understands her condition and possesses intact decision making capacity, her request should be honored. This position stems from respect for autonomy, and is supported by law in many states that recognize a competent patient's right to refuse treatment.

When is it justified for me to withhold the truth from a patient?

If the physicians has compelling evidence that disclosure will cause real and predictable harm, truthful disclosure may be withheld. Examples might include disclosure that would make a depressed patient actively suicidal. This judgment, often referred to as the "therapeutic privilege," is important but also subject to abuse. Hence it is important to invoke this only in those instances when the harm seems very likely, not merely hypothetical. If the patient him- or herself states an informed preference not to be told the truth. Some patients might ask that the physician instead consult family members, for instance. In these cases, it is critical that the patient give thought to the implications of abdicating their role in decision making. If they chose to make an informed decision not to be informed, however, this preference should be respected.

Is it ever appropriate to do a procedure for the first time without supervision?

If you have never done either of these activities before, it is your responsibility to ask for appropriate supervision before beginning the procedure. Emphasize your interest in learning the new skill as well as your interest in learning it under the best conditions possible.

I'm noticing what looks like addictive behavior in one of my classmates. What should I do?

Impaired students become impaired physicians. You are entering a profession that carries an obligation to its members for self-regulation. As a student, your classmate has an opportunity to seek help before serious harm comes to himself or herself, or to one of his or her patients. Once licensed, you will have a legal obligation to report colleagues to the medical board if they are "unable to practice medicine with reasonable skill and safety to patients by reason of illness, drunkenness, excessive use of drugs, narcotics, chemicals, or any other type of material, or as a result of any mental or physical conditions (

how pervasive is religiosity in canada

In 2019, 68% of Canadians aged 15 and older reported having a religious affiliation. In addition, just over half of Canadians (54%) said they considered their religious or spiritual beliefs to be somewhat or very important to how they live their lives. Participation in group or individual religious activities was less common: 23% of Canadians said they participated in a group religious activity at least once a month, and 30% said they engaged in a religious or spiritual activity on their own at least once a week.

Is it justifiable to withhold or withdraw food or fluids?

In BC it is legal for: Competent or incompetent patient and Oral or artificial nutrition and hydration This question underscores the importance of clarifying the goals of medical treatment. Any medical intervention can be withheld or withdrawn, including nutrition and IV fluids. At all times, patients must be given basic humane, compassionate care. They should be given a comfortable bed, human contact, warmth, and be kept as free from pain and suffering as possible. While some believe that food and fluids are part of the bare minimum of humane treatment, both are still considered medical treatments. Several court cases have established that it is justifiable to withhold or withdraw food and fluids.

Indigenous children in foster care

In Canada, 53.8% of children in foster care are Indigenous, but account for only 7.7% of the child population according to Census 2021. Results from the 2011 National Household Survey also show that 38% of Indigenous children in Canada live in poverty, compared to 7% for non-Indigenous children.

How does "mature minor consent" relate to immunization?

In all of these settings, a child can consent to the vaccine on their own behalf if the health care provider has determined that the child is capable of making this decision. Although there is no set age for when a child becomes capable, common practice is for parents or guardians of children 12 years of age and younger to give consent for their child to be immunized. However, there may be extenuating circumstances in which a child of this age may provide their own consent. Older children are given the opportunity to consent for themselves. If a child refuses a vaccine for which their parent or guardian has consented, they must be informed of the risks of not having it. The immunization records of any child who gives their own consent will not be shared with the parent or guardian, unless the child gives permission.

In caring for a person who is dying, knowing what would make the experience of dying__________is an important goal

In caring for a person who is dying, knowing what would make the experience of dying "good" is an important goal I find it doesn't take fancy techniques-you just need to be sincere and patient and interested. Listen more and talk less. Try asking something like, "Knowing that all of us have to think about dying at some point, what would be a good death for you?" What people choose when they think about a good death for themselves is often beyond what medicine can provide-for instance, an affirmation of love, a completion of important work, or a last visit with an important person. As a physician, I can't always make those things happen. But I can help the dying person get ready-and in this way, contribute to a death that is decent.

How do I decide whether to tell a patient about an error?

In general, even trivial medical errors should be disclosed to patients. Any decision to withhold information about mistakes requires ethical justification. If a physician believes there is justification for withholding information about medical error from a patient, his judgment should be reviewed by another physician and possibly by an institutional ethics committee. The physician should be prepared to publicly defend a decision to withhold information about a mistake from the patient

What if the patient starts to cry while I am talking?

In general, it is better simply to wait for the person to stop crying. If it seems appropriate, you can acknowledge it ("Let's just take a break now until you're ready to start again") but do not assume you know the reason for the tears (you may want to explore the reasons now or later). Most patients are somewhat embarrassed if they begin to cry and will not continue for long. It is nice to offer tissues if they are readily available (something to plan ahead); but try not to act as if tears are an emergency that must be stopped, and don't run out of the room--you want to show that you're willing to deal with anything that comes up.

Would a physician ever be justified in breaking a law requiring mandatory reporting?

In general, mandatory reporting requirements supersede the obligation to protect confidentiality. While the physician has a moral obligation to obey the law, she must balance this against her responsibility to the patient. Reporting should be done in a manner that minimizes invasion of privacy, and with notification to the patient. If these conditions cannot be met, or present an intolerable burden to the patient, the physician may benefit from the counsel of peers or legal advisors in determining how best to proceed

Is it justifiable to deceive a patient with a placebo?

In general, the deceptive use of placebos is not ethically justifiable. Specific exceptions should be rare and only considered if the following conditions are present: the condition is known to have a high placebo response rate the alternatives are ineffective and/or risky the patient has a strong need for some prescription

An elderly man who lives in a nursing home is admitted to the medical ward with pneumonia. He is awake but severely demented. He can only mumble, but interacts and acknowledges family members. The admitting resident says that treating his pneumonia with antibiotics would be "futile" and suggests approaching the family with this stance. Do you agree?

In many cases, "futility" is used inaccurately to describe situations that appear undesirable. For this patient, treating pneumonia with antibiotics stands a reasonable chance of success. The patient's quality of life, though low, is not unacceptably so. Unless the patient (or if found incapacitated, his surrogate) were to say that he would find this quality of life unacceptably low, there is neither quantitative nor qualitative grounds for calling antibiotics futile in this case. Unlike Cases 1 and 2, in Case 3 there is a treatment available that benefits the patient.

When is it appropriate to question a patient's ability to participate in decision making?

In most cases, it is clear whether or not patients have capacity to make their own decisions.

Are VSED and VSPeC as a path to MAiD legal in Canada?

In order to be eligible for MAiD, a person's natural death must have become reasonably foreseeable. VSED and VSPeC make a person's death reasonably foreseeable (indeed certain). The question is really whether a person's reasonably foreseeable death caused by VSED or VSPeC can be considered natural. This is a complicated question to answer: no court has ruled on it; no legislation directly addresses it; there are no clinical practice guidelines about it. It has been directly addressed in one disciplinary case, a complaint against a British Columbia doctor who determined that a patient's natural death had become reasonably foreseeable after refusing food and liquids. The committee hearing the complaint said: [the patient met the criteria for MAiD] despite the fact that her refusal of medical treatment, food, and water undoubtedly hastened her death and contributed to its 'reasonable foreseeability.' It has also been addressed indirectly in one of the two Charter challenges to Canada's MAiD legislation. In Lamb v. Canada, the Attorney General accepted as uncontested expert evidence (indeed, from their own expert) that: If Ms. Lamb were to be assessed now, and she indicated an intent to stop BiPaP [a machine similar to a CPAP machine, which helps a person breathe better while sleeping] and refuse treatment when she next developed pneumonia, it is likely that she would be found to meet the threshold for having a reasonably foreseeable natural death ... She would not be required to develop an episode of pneumonia before being approved for MAID. Most would consider it sufficient that she expresses certain intent to refuse treatment when this occurs, as she will inevitably develop a chest infection in the near future. This suggests that "reasonably foreseeable" is met if a person has demonstrated a clear intent to take steps (whether that be refusing medical treatment, preventive care, or food and liquids) that will make their death happen soon and of a predictable cause. Both the BC discipline case and the Lamb case suggest that VSED and VSPeC are legal paths to MAiD.

What should I know about the hospice approach?

In order to help someone towards a decent, or even good, death, the hospice framework is very helpful. Hospice started as a grassroots effort, as a view of dying that lets go of the possibility of cure. Instead, hospices emphasize symptom control and attention to psychological and spiritual issues. Pathophysiology becomes less important and personal meaning becomes more important. Thus this framework analyzes a person's medical care into four major topics, and this can be used to outline day-to-day care plans for a patient: Pain - one of the things most feared by patients with life-threatening illness. Symptom control - including dyspnea, nausea, confusion, delirium, skin problems, and oral care. Psychological issues - especially depression, sadness, anxiety, fear, loneliness. Spiritual or existential issues - including religious or non-religious beliefs about the nature of existence, the possibility of some type of afterlife.

UBC Vision

In pursuing this mission, we will emphasize open communication and ethical behavior, recognizing that it is a privilege to provide responsible and innovative stewardship of human, financial and all other resources.

How do I know if the treatment is no longer "of benefit?"

In some cases, the treatment may be "futile"; that is, it may no longer fulfill any of the goals of medicine. In general, these goals are to cure if possible, or to palliate symptoms, prevent disease or disease complications, or improve functional status. For example, patients with severe head trauma judged to have no chance for recovery of brain function can no longer benefit from being maintained on a mechanical ventilator. All that continuation would achieve in such a case is maintenance of biologic function. In such a case, it would be justifiable to withdraw mechanical ventilation.

What ethical issues are associated with CAM in clinical practice?

In the context of clinical practice, the ethical issues pertain to providing optimal medical care to an individual. Any physician, allopathic or otherwise, is bound by oath to do no harm and to provide the most efficacious therapies to their patient. The precepts of evidence-based medicine (EBM) and the accessibility of medical research literature provide clinicians with powerful tools to identify such therapies. In evaluating the risk of harm and the potential benefits of any therapy, weight must be given to the amount and quality of research that has been done on the intervention, known risks and side effects of the therapy, the credential and competence of the practitioner, the seriousness of the condition being treated, and the belief system and wishes of the patient. Given the relative dearth of research literature on many CAM therapies, the clinician must use best judgment to decide which therapies are unlikely to do harm, either directly or by reducing the effectiveness of other therapies, and which may offer some, if not great, benefit. A CAM therapy that is neither harmful nor effective can become damaging if it precludes the patient obtaining effective treatment. The advantage of integrative medical clinics is to take much of the guesswork out of this algorithm. Well-trained, licensed CAM providers working alongside conventional clinicians create an environment in which patient care can utilize current best practices in each discipline. CAM therapies for some medical conditions require close supervision by a CAM professional. Just as a patient with diabetes (or hypertension, depression, etc) should receive ongoing medical supervision for his/her diabetes (hypertension, depression, etc) management, the same holds for CAM therapies. Many warnings about the risks associated with CAM therapy use are grounded in an assumption that the patient may be self-treating and/or not receive adequate monitoring by a trained CAM professional.

Your patient has been suffering from chronic low back pain for many years now. She voices her frustration with the various treatment modalities that you have been trying and says she is considering getting acupuncture. How do you respond?

In this case, there are few clearly effective treatments for the medical condition. Hence, complementary approaches may be a reasonable recommendation, assuming they are not harmful. While you may know little about acupuncture yourself, you may encourage your patient to consult with local experts to inform herself about what acupuncture can offer. Encourage your patient to stay in contact with you and inform you about both the perceived benefits and any side effects she may experience.

How do physicians who care for the dying deal with their own feelings?

It is not hard to find physicians who are burned out - ask any nurse. What is difficult is to find for yourself a type of self-care that will enable you to develop your gifts as a physician, and continue to use them in practice. It helps to learn your strengths and weaknesses, and to actively seek whatever will nurture you - in or out of medicine. A strategy of detachment may not serve you well in the long run. There are indeed rewards for physicians who care for the dying, but as a Zen master once observed of a bingo game, "you must be present to win."

public health is distinct from medicine in several ways

It is oriented toward the health of populations or groups rather than individuals; the prevention, rather than the treatment, of disease; the elimination of systematic inequalities in population health associated with social class, race/ethnicity, gender, and other markers of disadvantage; and the use of state power and collective resources to accomplish these ends. to pursue these goals, public health must address a broad array of social conditions that extend beyond medical services. The "social determinants of health"-sometimes simply referred to as the conditions in which people grow up, live, work, and play (Commission on the Social Determinants of Health 2008)-include the built and physical environments (e.g., sidewalks, highway safety, food access, water and air quality); conditions of housing, schools, neighborhoods, and the workplace; and broader social, economic, and cultural conditions related to education and employment, income and wealth, racism and sexism, and social support and security

Is it justifiable to withhold or withdraw care because of costs?

It is rarely justifiable to discontinue life-sustaining treatment for cost reasons alone. While we should always try to avoid costly treatments that offer little or no benefit, our obligation to the patient outweighs our obligation to save money for health care institutions. There are rare situations in which costs expended on one terminally ill patient could be clearly better used on another, more viable patient. For instance, a terminally ill patient with metastatic cancer and septic shock is in the last ICU bed. Another patient, young and previously healthy, now with a self-limited but life-threatening illness, is in the emergency room. In such cases, it may be justifiable to withdraw ICU treatment from the terminally ill patient in favor of the more viable one. Even so, such decisions must be carefully considered, and made with the full knowledge of patients and their surrogate decision makers.

How should I respond when an intern asks if I want to practice a procedure on a patient who just died?

It remains your responsibility to assure that your interactions with the cadaver are respectful and only as invasive as necessary.

the power of public health as an agent of the state to restrict individual choice in efforts to prevent disease and promote health.

Many public health activities try to influence individual actions, though they may do so in more or less restrictive ways. Public health policy may eliminate choice altogether through, for example, compulsory quarantine of patients with infectious disease; restrict choice by, for example, banning smoking in public places or fluoridating public water supplies; guide choice through disincentives (e.g., taxes on health-harming goods, such as sugary beverages) and incentives (e.g., tax breaks on health-promoting goods); or inform choice through, for example, food labeling or media campaigns The analysis of which of these actions are or are not ethically and politically justifiable is often informed by theharm principle,

Safeguards - Natural Death Is Not Reasonably Foreseeable

Medical assistance in dying can only be provided to persons who can give consent. Consent through an alternate or substitute decision maker or through a personal advance directive is not applicable. A person's request for medical assistance in dying must be made in writing and signed and dated in front of one "independent witness" (see below), who must also sign and date the request.If a person can provide informed consent, but is physically unable to sign a request, another person may sign the request under the person's express direction (a "Proxy"). A person's request must be signed and dated after they have been informed by a doctor or nurse practitioner that they have a medical condition that fits the "grievous and irremediable" criteria. Two independent doctors or nurse practitioners must assess the person to confirm their eligibility, one of whom must have expertise in the person's illness, disease or disability.If neither of the two assessing doctors or nurse practitioners has expertise in the illness, disease, or condition causing the person's suffering, one of the assessors must consult with a doctor or nurse practitioner who does have this expertise, and must share the results of that consultation with the other assessor. The two assessing doctors or nurse practitioners must ensure that all reasonable means available to relieve the person's suffering have been discussed and, furthermore, that the person has given serious consideration to these means. A person must be given a period of reflection of at least 90 days from the date of their first eligibility assessment to the date that medical assistance is provided, unless both assessors agree that the person is at risk of losing their ability to consent and agree that a shorter period is appropriate under the circumstances. A person must be given the opportunity to withdraw their request throughout the process, including immediately before being provided with medical assistance in dying.

Safeguards - Natural Death Is Reasonably Foreseeable

Medical assistance in dying can only be provided to persons who can give consent. Consent through an alternate or substitute decision maker or through a personal advance directive is not applicable. A person's request for medical assistance in dying must be made in writing and signed and dated in front of one "independent witness" (see below), who must also sign and date the request.If a person can provide informed consent, but is physically unable to sign a request, another person may sign the request under the person's express direction (a "Proxy"). A person's request must be signed and dated after they have been informed by a doctor or nurse practitioner that they have a medical condition that fits the "grievous and irremediable" criteria. Two independent doctors or nurse practitioners must assess the person to confirm their eligibility. A person must be given the opportunity to withdraw their request throughout the process, including immediately before being provided with medical assistance in dying.

When can parental authority to make medical decisions for their children be challenged?

Medical caretakers have an ethical and legal duty to advocate for the best interests of the child when parental decisions are potentially dangerous to the child's health, imprudent, neglectful, or abusive. As a general rule, medical caretakers and others should challenge parental decisions when those decisions place the child at significant risk of serious harm. When satisfactory resolution cannot be attained through respectful discussion and ethics consultation, seeking involvement of a State child protection agency or a court order might be necessary. However, parental decision-making should be guided by the best interests of the child. Decisions that are clearly not in a child's best interest can and should be challenged.

The goals of advance care planning

Minimize the burden of decision making on the spokesperson and/or family members. Reduce the likelihood of conflicts between a patient's spokesperson, family members and health care providers, and Minimize the likelihood of over- or under-treatment, Maximize the likelihood that medical care serves the patient's goals These goals reflect respect for the principles of patient autonomy (right to self-determination in light of personal interests including goals, preferences, and concerns for one's family), beneficence (promoting good) and non-maleficence (avoiding harm)

A better word for "competent":

"decision making capacity"

in BC new eligibility criteria for providing medical assistance in dying to those who have a :

"grievous and irremediable condition," but whose natural death is not reasonably foreseeable.

The fundamental principles of professionalism are stated as

1) the primacy of patient welfare 2) patient autonomy 3) social justice. Professional responsibilities that follow from these principles are commitment to competence, to honesty with patients, to confidentiality, to appropriate relationship with patients, to improving quality of care, to improving access to care, to a just distribution of finite resource, to scientific knowledge, to maintaining trust by managing conflicts of interests and to professional responsibilities.

What if I see my resident or attending doing something "unethical"?

Ideally, you could talk with your resident about what you observed. Everyone has a unique perspective and your resident may have a rationale for his behavior that was unknown to you. Approaching him honestly, with simple questions, may allow him the benefit of the doubt and open up a dialogue between you. The nature of the observed 'unethical' act determines what your obligations are. In simpler cases, it can be a matter of treating it as a negative lesson in how NOT to be a physician. In more complex instances, patient care may be in jeopardy and you may have an obligation to report the resident's behavior if he refuses to discuss it with you directly. Your attending physician or clerkship coordinator can be valuable resources as you make these judgment calls. Discussing these instances with your peers can also be helpful.

Where can you go in BC if you need guidance on confidentiality

Legislative Guidance, "Duty to Report" Registrants may seek guidance on these issues by contacting the College or by seeking medical legal advice from the CMPA (The Canadian Medical Association (CMA) Code of Ethics and Professionalism)

What is the difference between a futile intervention and an experimental intervention?

Making a judgment of futility requires solid empirical evidence documenting the outcome of an intervention for different groups of patients. Futility establishes the negative determination that the evidence shows no significant likelihood of conferring a significant benefit. By contrast, treatments are considered experimental when empirical evidence is lacking and the effects of an intervention are unknown.

Positive impacts of the AIDS epidemic

On the federal level AIDS activists forced the more rapid approval of medicines by the Federal Food and Drug Agency (FDA). State and city departments of public health had to organize culturally sensitive, anonymous HIV counseling and testing centers individual practice level, physicians were forced to confront their own biases to provide ongoing care for a new and possibly transmissible epidemic.

Why is medical futility controversial?:

One source of controversy centers on the exact definition of medical futility, which continues to be debated in the scholarly literature. Second, an appeal to medical futility is sometimes understood as giving unilateral decision-making authority to physicians at the bedside. Proponents of medical futility reject this interpretation, and argue that properly understood futility should reflect a professional consensus, which ultimately is accepted by the wider society that physicians serve. Third, in the clinical setting, an appeal to "futility" can sometimes function as a conversation stopper. Thus, some clinicians find that even when the concept applies, the language of "futility" is best avoided in discussions with patients and families. Likewise, some professionals have dispensed with the term "medical futility" and replaced it with other language, such as "medically inappropriate." Finally, an appeal to medical futility can create the false impression that medical decisions are value-neutral and based solely on the physician's scientific expertise. Yet clearly this is not the case. The physician's goal of helping the sick is itself a value stance, and all medical decision making incorporates values

the fiduciary nature of the clinician-patient relationship

The provider has knowledge, influence, and power in the relationship, which entail special responsibilities.

Are advance requests for MAiD legal in Canada?

There are two categories of advance requests that are possible under Canadian law. "Advance consent" is available to individuals in Tracks One or Two in the context of failed self-administration of MAiD. A person can enter into a written arrangement to be given provider-administered MAiD if they have not died after a specified period of time after self-administration and lack decision-making capacity. "Final consent waiver" is only available to individuals on Track One (i.e., their natural death has become reasonably foreseeable). A person who has met all of the eligibility criteria for MAiD and whose natural death has become reasonably foreseeable may enter into a written agreement with their medical or nurse practitioner to be provided MAiD on a specific day in the future should they have lost the capacity to consent to MAiD by or on that date.

DNR-What if patients are unable to express what their wishes are?

These approaches include Advance Care Planning and the use of surrogate decision makers.

Talking to family members about their parents AD/ MOST:

Treatments she would want to have or any abilities she wouldnt want to live without Has she ever spoke out to you on what were to happen if she got sick "if your mom could speak right now, what do you think she would say about her medical care wishes?" "how much do you think she would go through for the possibility of more time?" reduce the uncertainity and stress that your family may face if they need to make decisions on your behalf

Consent by minors is referred to as

assent

Futility refers to a particular intervention

at a particular time, for a specific patient Futility does not apply to treatments globally, to a patient, or to a general medical situation For example, rather than stating, "It is futile to continue to treat this patient," one would state, "CPR would be medically futile for this patient."

why take relegion into the question?

caring implies care for the whole person, physically, emotionally, socially and spiritually. Thus, our concern for the patient's spiritual well-being is an integral part of health care and is a way of showing respect for the person who comes as a patient-supplicant. not underestimate the importance of the patient's belief system.

The proxy directive

generally a Durable Power of Attorney for Health Care (DPAHC), allows for the designation of a spokesperson or surrogate medical decision maker of the patient's choosing. This surrogate decision maker (often referred to as a health care agent) makes health care decisions for the patient in the event she is incapacitated.

As a trainee, should I do advance care planning with my patients?

medical students and residents should engage the patients they are caring for in these discussions. If the patient has been recently diagnosed with a terminal or life changing condition, has severe depression, demonstrates paranoid ideation, or is suicidal, you should ask the responsible attending physician whether this is an appropriate time to raise these issues. Otherwise, you should initiate the discussions and request faculty support (such as role modeling or mentoring) if needed. You should review the framing of the discussion and the patient's views with the attending physician responsible for the patient's overall care.

A 3-year-old child is brought to your clinic with a fever and stiff neck. You are quite certain the child has meningitis. When you discuss the need for a spinal tap and antibiotic treatment, the parents refuse permission, saying, "We'd prefer to take him home and have our minister pray over him." Can the parents refuse treatment in this case? How should you handle this?

The physician has a duty to challenge the decision of parents when their refusal of treatment would pose a significant risk of substantial harm. Failure to diagnose and treat bacterial meningitis would seriously threaten the health and even life of this child. The physician should share his or her view with the family and seek to elicit their cooperation through respectful discussion. The physicians should be open to alternatives that satisfy the parents' concerns and achieve the goal of keeping the child safe. Inviting the family's religious leader to the hospital while also providing standard medical therapy may prove to be an acceptable compromise. Should these efforts not result in parental permission, the physician is justified in seeking legal authority (in the form of a court order of authorization from a state child protection agency) to proceed with the procedure and treatment of the child. In most states a physician is legally authorized to provide emergency treatment to a child without a court order when delay would likely result in harm. *check BC

A mother brings her 18-month-old daughter to your office for a routine physical examination. The child has had no immunizations. Her mother says that they believe that vaccines weaken the immune system and have heard that vaccination can cause autism. What is your role in this situation? Can parents refuse to immunize their children?

The risk faced by unimmunized individuals is relatively low, and the mother's refusal to immunize does not pose a significant likelihood of serious harm to her child. The physician should be sure that the child's mother understands the risks of remaining unimmunized and attempt to correct any misconceptions about the degree of risk associated with getting immunized. If the mother persists in her request, the physician should respect her wishes.

What happens when an older child disagrees with her parents about a medical treatment?

The wishes of competent older children regarding their medical care should be taken seriously. If the medical caretaker judges a child competent to make the medical decision in question, she should first attempt to resolve the issue through further discussion. If that fails, the medical caretaker should assure that the child's voice has been heard and advocate for the child. In intractable cases, an ethics consultation or judicial hearing should be pursued. younger children Most children and adolescents lack full capacity to make complex medical decisions, however, and final authority to make medical decisions will usually remain with their parents.

Jordan River Anderson

This is because different levels of government fund different services for First Nations children, especially those living on-reserve. This has led to disputes between governments about who should pay for which services. Jordan River Anderson from Norway House Cree Nation in Manitoba got caught in one of these payment disputes. As a result, he didn't get the recommended home-based care he needed. Jordan was born in 1999 with multiple disabilities and stayed in the hospital from birth. When he was 2 years old, doctors said he could move to a special home for his medical needs. However, the federal and provincial governments could not agree on who should pay for his home-based care. Jordan stayed in the hospital until he passed away at the age of 5. In 2007, the House of Commons passed Jordan's Principle in memory of Jordan. It was a commitment that First Nations children would get the products, services and supports they need, when they need them. Payments would be worked out later. Today, Jordan's Principle is a legal obligation, which means it has no end date. While programs and initiatives to support it may only exist for short periods of time, Jordan's Principle will always be there. Jordan's Principle will support First Nations children for generations to come.

How do you know when someone is dying?

This question is not as simple as it might sound. The SUPPORT study demonstrated that even for patients with a high probability of dying, it is still difficult for a clinician to predict that a particular patient is about to die. Thus it may be more useful for clinicians to give up relying on their predictive skills, and look at the common clinical paths (or trajectories) taken by dying patients, and design medical care that includes "contingency plans" for clinical problems that a person with incurable lung cancer (for example) is likely to experience. Such contingency plans might include advance directives and perhaps DNAR orders, as well as lines such as: "You will probably die from this, although we can't predict exactly when. What is really important for you in the time you have left?"

Does the patients have to be terminally ill to refuse treatment?

Though in most cases of withholding or withdrawing treatment the patient has a serious illness with limited life expectancy, the patient does not have to be "terminally ill" in order for treatment withdrawal or withholding to be justifiable.

T/F It may be appropriate to continue temporarily to make a futile intervention available

True In some instances, it may be appropriate to continue temporarily to make a futile intervention available in order to assist the patient or family in coming to terms with the gravity of their situation and reaching closure. For example, a futile intervention for a terminally ill patient may in some instances be continued temporarily in order to allow time for a loved one arriving from another state to see the patient for the last time. However, futile interventions should not be used for the benefit of family members if this is likely to cause the patient substantial suffering, or if the family's interests are clearly at odds with those of the patient.

When is it appropriate for a physician to recommend a specific course of action or override patient preferences?

Under certain conditions, a physician should strongly encourage specific actions. When there is a high likelihood of harm without therapy, and treatment carries little risk, the physician should attempt, without coercion or manipulation, to persuade the patient of the harmful nature of choosing to avoid treatment. Court orders may be invoked to override a patient's preferences. However, such disregard for the patient's right to noninterference is rarely indicated. Court orders may have a role in the case of a minor; during pregnancy; if harm is threatened towards oneself or others; in the context of cognitive or psychological impairment; or when the patient is a sole surviving parent of dependent children. However, the use of such compulsory powers is inherently time-limited, and often alienates the patient, making him less likely to comply once he is no longer subject to the sanctions.

Close in age exceptions

A 14 or 15 year old can consent to sexual activity as long as the partner is less than five years older and there is no relationship of trust, authority or dependency or any other exploitation of the young person. This means that if the partner is 5 years or older than the 14 or 15 year old, any sexual activity is a criminal offence. There is also a "close in age" exception for 12 and 13 year olds. A 12 or 13 year old can consent to sexual activity with a partner as long as the partner is less than two years older and there is no relationship of trust, authority or dependency or any other exploitation of the young person. This means that if the partner is 2 years or older than the 12 or 13 year old, any sexual activity is a criminal offence.

When does a fetus or a newborn become a person? Does a fetus have rights?

A child becomes a human being within the meaning of this Act when it has completely proceeded, in a living state, from the body of its mother, whether or not (a) it has breathed; (b) it has an independent circulation; or (c) the navel string is severed. Further, the Supreme Court has stated that a woman and her fetus are considered "physically one" person under the law. Fetuses do have a limited form of retroactive rights in Canada via the "born alive" rule. While the fetus has no independent legal status in utero, once born alive, the child can enforce its legal rights retroactively to address events that occurred during its gestation, such as a physical injury. However, a child cannot seek redress from its own mother for any action she took during the pregnancy.12 The "born alive" rule may be most relevant in estate law - i.e., you can leave an inheritance to a fetus, who is legally entitled to it only once it's born alive.

How does informed consent apply to children?

A child who is assessed by a health care provider as being capable to give consent is called a "mature minor". A child who is a mature minor may make their own health care decisions independent of their parents' or guardians' wishes. In B.C. there is no set age when a child is considered capable to give consent. A health care provider can accept consent from the child and provide health care that is in the child's best interests without getting consent from the parent or guardian if the health care provider is sure that the child understands: The need for the health care What the health care involves and The benefits and risks of the health care The primary responsibility of the physician is the well-being of the child. Therefore, if the parental decision places the child at risk of harm then further action may be indicated. When there are differences in opinion between the parents and physicians that cannot be resolved ethics consultation may be pursued, and legal avenues may be pursued when all other means have failed. Children should be included in decision-making at a developmentally appropriate level and assent should be sought when possible.

Example conversation for Medical Orders for Scope of Treatment (MOST)

A document that helps you communicate your health care wishes Im happy to see that you... I think it may be a good time to talk about where things are at, and where things are going. are you comfertable about talking abotu thsi now, or woudl you perfer that your daughter also be here? Youre health has changed quite a bit over the past few months, what is your understanding of your illiness? And your overall health "that must be difficult for you________ (insert name)" I am worried that you may have another health scare, and time may be short if that should happen, it is important that your family and health care team understand your goals of care, we need to know what is most important to you, what do you hope the treatments will achieve. we all want to be clear on your wishes, and help you make plans that fit your desires "i know this must be a lot to take in after what you have been through"

What is public health?

What we, as a society, do collectively to assure the conditions in which people can be healthy The science and art of preventing disease, prolonging life and promoting health through organized efforts of society A movement dedicated to the equitable improvement of health and wellbeing of communities with their full participation. Public health is the science of protecting and improving the health of communities through education, promotion of healthy lifestyles, and research for disease and injury prevention The key functions of public health agencies are assessing community health needs and marshalling the resources for responding to them, developing health policy in response to specific community and national health needs, and assuring that conditions contribute to good health, including high quality medical services, safe water supplies, good nutrition, unpolluted atmospheres and environments that offer opportunities for exercise and recreation are available to individuals [T]he promotion of health and the prevention of disease and disability; the collection and use of epidemiology data, population surveillance, and other forms of empirical quantitative assessment; a recognition of the multidimensional nature of the determinants of health; and a focus on the complex interactions of many factors-biological, behavioral, social and environmental-in developing effective interventions

When can CPR be withheld in the OR?

When CPR is judged to be of no benefit to the patient. When a patient with intact decision-making capacity (or in the case of those without decision making capacity, an appropriate surrogate decision-maker) indicates that they do no want CPR, even if the need arises.

When can CPR be withheld?

When CPR will likely be ineffective and has minimal potential to provide direct medical benefit to the patient. When the patient with intact decision making capacity or a surrogate decision maker explicitly requests to forgo CPR.

What if parents are unavailable and a child needs medical treatment?

When parents are not available to make decisions about a child's treatment, medical caretakers may provide treatment necessary to prevent harm to the child's health. In general, a child can be treated or transported without parental permission if the child has an emergency condition that places his or her life or health in danger, the legal guardian is unavailable or unable to provide permission for treatment or transport, and treatment or transport cannot be delayed without further endangering the child. Providers should administer only those treatments necessary to prevent harm to the child until parental permission can be obtained. Examining a child who presents to medical attention is always appropriate in order to establish whether a threat of life or health exists.

What happens when a patient wants to include treatment preferences that fall outside standard practices in medicine (e.g., herbal remedies, medically futile treatments

When these occur, they provide opportunities to negotiate about culturally sensitive approaches to health care and establish limits to patient autonomy based on professional standards.

How does the health care provider decide if a child is capable to give consent for immunizations?

Which vaccine they are due to get The benefits of getting immunized The risk of not getting immunized Common and expected side effects Rare serious side effects Medical reasons to not receive a vaccine The health care provider will give the child time to read over the information and allow the child to ask questions. The child will then be asked some questions to make sure they understand the information and are ready to make a decision. If the health care provider is not sure that a child understands, or a child is not ready to make a decision, the child will not be immunized.

Bill C-92 - an Act Respecting First Nations, Métis and Inuit Youth and Families

affirms the rights and jurisdiction of Indigenous peoples in relation to child and family services and sets out principles applicable, on a national level, to the provision of child and family services in relation to Indigenous children, such as the best interests of the child, cultural continuity and substantive equality. Co-developed with Indigenous, provincial and territorial partners, the act: affirms the rights of First Nations, Inuit and Métis peoples to exercise jurisdiction over child and family services establishes national principles such as the best interests of the child, cultural continuity and substantive equality contributes to the implementation of the United Nations Declaration on the Rights of Indigenous Peoples provides an opportunity for Indigenous peoples to choose their own solutions for their children and families On June 21, 2019, the Act respecting First Nations, Inuit and Métis children, youth and families became an official law, and on January 1, 2020, its provisions came into force.

Other students have (unauthorized) access to last year's killer exam. Should I look at it?

cheating in class is an example of unprofessional behavior. It represents a lack of fairness, lack of integrity, and can foreshadow lying in other contexts during your medical training. As a member of a profession, you are accountable for your own behavior and for the behavior of your colleagues. The Assistant Dean for Student Affairs or the Medical Student Association (MSA) representative can field your concerns and help you develop a plan for confronting your classmates.

On January 4, 2022, the Government of Canada and the parties to the complaint before the Canadian Human Rights Tribunal (CHRT) regarding the First Nations Child and Family Services program and Jordan's Principle, and the parties to 2 Federal Court class actions announced that agreements-in-principle were reached on a global resolution for:

compensation for First Nations children on reserves and in Yukon who were removed from their homes, and those impacted by the government's narrow definition of Jordan's Principle, including for their parents and caregivers long-term reform of the First Nations Child and Family Services program and a renewed approach to Jordan's Principle, in order to eliminate discrimination and to provide reform to Indigenous Services Canada to prevent the discrimination from recurring

In order for the patient's consent to be valid, she must be considered:

competent to make the decision at hand and her consent must be voluntary. It is easy for coercive situations to arise in medicine. Patients often feel powerless and vulnerable. To encourage voluntariness, the physician can make clear to the patient that she is participating in a decision-making process, not merely signing a form. With this understanding, the informed consent process should be seen as an invitation for the patient to participate in health care decisions. The physician is also generally obligated to provide a recommendation and share his reasoning process with the patient. Comprehension on the part of the patient is equally as important as the information provided. Consequently, the discussion should be carried on in layperson's terms and the patient's understanding should be assessed along the way.

allopathic medicine

conventional western medicine

To be eligible for medical assistance in dying, a person must be:

diagnosed with a grievous and irremediable medical condition and experience intolerable suffering

How can disagreements on the multidisciplinary team be handled?

disagreement leads to a more complete interprofessional discussion of the patient's care, resulting in a new consensus about the best course of action. The new consensus may require compromises from each individual. When members of a team cannot arrive at a consensus of what should be done, it may be helpful to consult other professionals who are not directly involved in the patient's care team for objective input. If the disagreement still cannot be resolved, another resource may be the hospital's ethics committee, which can listen to disagreements and help suggest solutions.

Informed consent is the process by which the treating health care provider :

discloses appropriate information to a competent patient so that the patient may make a voluntary choice to accept or refuse treatment. It originates from the legal and ethical right the patient has to direct what happens to her body and from the ethical duty of the physician to involve the patient in her health care.

The patient's consent should only be "presumed," rather than obtained, in:

emergency situations when the patient is unconscious or incompetent and no surrogate decision maker is available, and the emergency interventions will prevent death or disability. In general, the patient's presence in the hospital ward, ICU or clinic does not represent implied consent to all treatment and procedures. The patient's wishes and values may be quite different from the values of the physician. While the principle of respect for person obligates you to do your best to include the patient in the health care decisions that affect her life and body, the principle of beneficence may require you to act on the patient's behalf when her life is at stake.

Futile interventions may:

increase a patient's pain and discomfort in the final days and weeks of life give patients and family false hope delay palliative and comfort care and expend finite medical resources.

Why dont Canadians have a family Dr

io Our population in Canada is growing -the population is also aging (more than double the doctors who retired in the pandemic) The new generation wants to work in teams. And we have not been building the teams where the new grads want to come into We are going to have to invest in primary care. We know that health systems that are built on good primary care have better health outcomes, better equity, and lower costs. Ex: you dont end up in hospital with stroke if your PCP has been monitoring your BP and treating it Building for teams and infrastructure that make the new generation want new generation of doctors that want to work in primary care Ex: when my patient on a bunch of medication gets covid and needs Paxlovid (which has a bunch of contradictions) i can pass this patient off to the pharmacist that is more equipped, more mastery with drug interactions to help. Allowing me to see the next patient

Advance care planning

is a process to help patients with decision-making capacity guide future healthcare decisions in the event that they become unable to participate directly in their care. Verbal directives may be ethically valid, but most patients and health care providers prefer written, official documentation. If official forms are not used, health care providers should document the result of their advance care planning conversations in a medical record progress note.

What are the physician's professional obligations with respect to CAM?

it is especially important to track the patient's use of all CAM therapies -Offhand dismissal or ridicule of CAM will impair communication and the therapeutic relationship with the patient; harmful herb-drug interactions could be missed or the patient may break entirely with the allopathic system. herb-drug interactions should be studied as rigorously as drug-drug interactions.

Good resources for guidance of Cross-Cultural Issues and Diverse Beliefs

patients and family members themselves, staff members with personal knowledge or experience, hospital chaplains, social workers, and interpreters. It is important to explore each individual's beliefs, as shared membership in a particular religious or cultural group does not necessarily entail identical belief systems.

How much do patients need to be told?

patients should be told all relevant aspects of their illness, including the nature of the illness itself, expected outcomes with a reasonable range of treatment alternatives, risks and benefits of treatment, and other information deemed relevant to that patient's personal values and needs. Treatment alternatives that are not medically indicated or appropriate need not be revealed. Facts that are not important to the patients ability to be an informed participant in decision making, such as results of specific lab tests, need not be told to the patient. Also, complete and truthful disclosure need not be brutal; appropriate sensitivity to the patient's ability to digest complicated or bad news is important.

Writing a DNR with your patient

physicians should discuss resuscitation preferences with the patient or his/her surrogate decision maker This conversation should be documented in the medical record, indicating who was present for the conversation, who was involved in the decision making process, the content of the conversation, and the details of any disagreement. These conversations are difficult and involve a careful consideration of the potential likelihood for clinical benefit within the context of the patient's preferences. Physicians can most effectively guide the conversation by addressing the likelihood of direct benefit from cardiopulmonary resuscitation within the context of the overall hopes and goals for the patient. They can then partner with the patient and his or her family to determine the clinical interventions that most effectively achieve these goals This approach is described by the palliative care literature as a goal oriented approach to providing end of life care.

Neonatal-Decisions regarding treatment options must be based on

population-based studies, physician recommendations, and parental perspectives.

Implicit in the understanding of the word suicide is the notion of a :

premature death that is being hastened out of despair, therefore when mental illness impairs judgment, intervention to stop a suicide is ethically warranted because the person seeking suicide has lost his ability to carefully weigh the benefits and burdens of continued life

What is the difference between a profession and a business?

professionals have a fiduciary duty toward those they serve. This means that professionals have a particularly stringent duty to assure that their decisions and actions serve the welfare of their patients or clients, even at some cost to themselves. Professions have codes of ethics which specify the obligations arising from this fiduciary duty. Ethical problems often occur when there appears to be a conflict between these obligations or between fiduciary duties and personal goals.

What does the duty of confidentiality require?

prohibits the health care provider from disclosing information about the patient's case to others without permission and encourages the providers and health care systems to take precautions to ensure that only authorized access occurs.

"Medical futility"

refers to interventions that are unlikely to produce any significant benefit for the patient.

What should be done when breaking confidentiality due to threat of themselves/others?

the child should be told why their information won't be kept private and who it will be given to. When appropriate, consider informing patients of your intention to disclose or report their personal health information to a third party in this context, and the information that will be shared. This is not necessary if you consider that doing so might pose a risk to yourself or others.

What happens when there is disagrement in interpretation or the authority of a patient's living will,

the medical team should meet with the family and clarify what is at issue. The team should explore the family's rationale for disagreeing with the living will. Do they have a different idea of what should be done (e.g., based on other communication from the patient)? Do they have a different impression of what would be in the patient's best interests, given her values and commitments? Or does the family disagree with the physician's interpretation of the living will? Is there a conflict of interest that may be fueling disagreement between the patient and family members? However, if the family merely does not like what the patient has requested, they do not have much ethical authority to sway the team. If the disagreement is based on new knowledge, substituted judgment, or recognition that the medical team has misinterpreted the living will, the family has much more say in the situation and most hospitals would defer to the family in these situations. If no agreement is reached, the hospital's Ethics Committee or Ethics Consultation Service should be consulted.

What if I see someone else make a mistake?

the observing physician has some obligation to see that the truth is revealed to the patient. This should be done in the least intrusive way. If the other health care provider does not reveal the error to the patient, the physician should encourage her to disclose her mistake to the patient. Should the health care provider refuse to disclose the error to the patient, the physician will need to decide whether the error was serious enough to justify taking the case to a supervisor or the medical staff office, or directly telling the patient. The observing physician also has an obligation to clarify the facts of the case and be absolutely certain that a serious mistake has been made before taking the case beyond the health care worker involved.

The most important goal of informed consent is that:

the patient has an opportunity to be an informed participant in her health care decisions.

Many warnings about the risks associated with CAM therapy use are grounded in an assumption that:

the patient may be self-treating and/or not receive adequate monitoring by a trained CAM professional.

If no appropriate surrogate decision maker is available:

the physicians are expected to act in the best interest of the patient until a surrogate is found or appointed. In rare circumstances, when no surrogate can be identified, a guardian ad litem may have to be appointed by the court. Confer with social work and risk management if you have trouble finding a legal surrogate for the patient.

Confidentiality- What if a family member asks how the patient is doing?

the requirements for making an exception to confidentiality may not be met. If there is not explicit permission from the patient to share information with family member, it is generally not ethically justifiable to do so. Except in cases where the spouse is at specific risk of harm directly related to the diagnosis, it remains the patient's (and sometimes local public health officers'), rather than the physician's, obligation to inform the spouse.

A 16 or 17 year old cannot consent to sexual activity if:

their sexual partner is in position of trust or authority towards them, for example their teacher or coach the young person is dependent on their sexual partner, for example for care or support the relationship between the young person and their sexual partner is exploitative The following factors may be taken into account when determining whether a relationship is exploitative of the young person: the young person's age the age difference between the young person and their partner how the relationship developed (for example, quickly, secretly, or over the internet) whether the partner may have controlled or influenced the young person

What obligations do clinicians have in difficult patient-clinician relationship

they should identify trusted colleagues with whom they can share their frustrations, employ strategies that allow the best in the relationship to prevail, and use a team approach. Clinicians should also try to address or manage their own attitudes and behaviors that contribute to the problem, recognizing that the patient's behaviors and attitudes may not change. The clinicians should do everything they can to maintain a therapeutic relationship (even one that is not ideal), however in some circumstances they may need to transfer care to another provider. This transfer can be done without threatening the patient. The treating provider can help to maintain the patient's trust in the health system, while also setting clear boundaries.

Comprehensive patient care often involves:

trying to solve problems which are beyond the scope of expertise and training of any one provider. Thus, the organization of professionals involved in one patient's care has evolved from that of a hierarchy, with the physician in a "command" position, to that of a multidisciplinary team, interfacing many different kinds of health care professionals, each with separate and important knowledge, technical skills, and perspective

What if the patient is not competent?

two general approaches to this dilemma: Advance Directives and surrogate decision makers. Advance Directive: This is a document which indicates with some specificity the kinds of decisions the patient would like made should he/she be unable to participate. In some cases, the document may spell out specific decisions (e.g. Living Will), while in others it will designate a specific person to make health care decisions for them (i.e. Durable Power of Attorney for Health Care). There is some controversy over how literally Living Wills should be interpreted. In some cases, the document may have been drafted in the distant past, and the patient's views may have changed. Similarly, some patients do change their minds about end-of-life decisions when they actually face them. In general, preferences expressed in a Living Will are most compelling when they reflect long held, consistently stable views of the patient. This can often be determined by conversations with family members, close friends, or health care providers with long term relationships with the patient. Surrogate decision maker:In the absence of a written document, people close to the patient and familiar with their wishes may be very helpful. (See Advance Care Planning.) The law recognizes a hierarchy of family relationships in determining which family member should be the official "spokesperson," though generally all close family members and significant others should be involved in the discussion and reach some consensus. The hierarchy is as follows: Legal guardian with health care decision-making authority Individual given durable power of attorney for health care decisions Spouse Adult children of patient (all in agreement) Parents of patient Adult siblings of patient (all in agreement)

In general, the capacity to make treatment decisions, including to withhold or withdraw treatment, is considered intact if the patient:

understands the clinical information presented appreciates his/her situation, including consequences with treatment refusal is able to display reason in deliberating about their choices is able to clearly communicate their choice. If the patient does not meet these criteria, then their decision to refuse treatment should be questioned, and handled in much the same way as discussed for the clearly incompetent patient. When in doubt, an ethics consultation may prove helpful.

How is MAiD in Canada different from MAiD in other parts of the world?

unlike anywhere else, nurse practitioners are allowed to provide MAiD unlike the American states, provider-administered MAiD is allowed unlike the American states, access to MAiD is not limited to those who are terminally ill unlike the European countries, whether suffering is intolerable is assessed entirely by the person

Compliance can be improved by:

using shared decision making. For example, physicians can say, "I know it will be hard to stay in bed for the remainder of your pregnancy. Let's talk about what problems it will create and try to solve them together." "I can give you a medication to help with your symptoms, but I also suspect the symptoms will go away if you wait a little longer. Would you prefer to try the medication, or to wait?" Or, "I understand that you are not ready to consider counseling yet. Would you be willing to take this information and find out when the next support group meets?" "Sometimes it's difficult to take medications, even though you know they are important. What will make it easier for you to take this medication?"

In situations where you believe an ethical or legal exception to confidentiality exists, ask yourself the following question:

will lack of this specific patient information put another person or group you can identify at high risk of serious harm? If the answer to this question is no, it is unlikely that an exception to confidentiality is ethically (or legally) warranted. The permissibility of breaching confidentiality depends on the details of each case. If a breach is being contemplated, it is advisable to seek legal advice before disclosure.

Is a DNR valid without an advance directive?

yes Although a DNAR order may be a component of an advance directive or indicated through advance care planning, it is valid without an advance directive.

Advance care planning process (4 steps)

1) thinking through one's relevant values and preferences 2) talking about one's values and preferences with one's spokesperson, close family members and health care providers 3) documenting them with an advance directive 4) reviewing them periodically and updating them as needed.

Instructive directives

Allow for preferences regarding the provision of particular therapies or classes of therapies. Living wills are the most common examples of instructive directives, but other types of instructive directives, such as no transfusion and no CPR directives are also employed.

Is it possible to predict which infants will not survive despite aggressive medical/surgical care in the neonatal period?

Although general risks can be identified, there are no successful models to assess individualizable predictability of death or disability. Illness severity scores (SNAP-II, CRIB scores) and health care provider intuition scores exist, but none have been validated to predict individual outcomes.

You are a 25-year-old female medical student doing a rotation in an HIV clinic. Sara is a 30-year-old woman with advanced HIV who dropped out of college after she found that she contracted HIV from her husband, who has hemophilia. In talking to Sara, it turns out you share a number of things--you are from the same part of Montana originally, also have young children, and like to cook. Later in the visit, when you suggest that she will need some blood tests, she gets very angry and says, "What would you know about this?" What happened?

Although the protocol for breaking bad news is helpful, it doesn't cover everything. There are instances when you may provoke a reaction from a patient because you remind them of someone else--or, as in this case, themselves. In these instances it can be helpful to step back, get another perspective (perhaps from someone in clinic who has known Sara), and try not to take this reaction too personally--even though it is likely that Sara will know how to really bother you.

What are the recognized obligations and values of a professional physician?

Altruism: A physician is obligated to attend to the best interest of patients, rather than self-interest. Accountability: Physicians are accountable to their patients, to society on issues of public health, and to their profession. Excellence: Physicians are obligated to make a commitment to life-long learning. Duty: A physician should be available and responsive when "on call," accepting a commitment to service within the profession and the community. Honor and integrity: Physicians should be committed to being fair, truthful and straightforward in their interactions with patients and the profession. Respect for others: A physician should demonstrate respect for patients and their families, other physicians and team members, medical students, residents and fellows.

Who can be an "independent witness" to a person's request for medical assistance in dying?

An "independent witness" can be any person who is at least 18 years of age and who understands the nature of the request, except if they: know or believe that they are a beneficiary under the will of the person making the request, or would receive a financial or other material benefit resulting from that person's death; are an owner or operator of any health care facility where the person making the request is being treated or lives; provide health care services or personal care to the person making the request (unless this is as their primary occupation for which they are paid); or are a doctor or nurse practitioner who is involved in assessing the person's eligibility for medical assistance in dying or providing the person with medical assistance in dying.

Difference between Physician Order for Life Sustaining Treatment (POLST) and an advanced directive

An advance directive is a direction from the patient, not a medical order. In contrast, a POLST form consists of a set of medical orders that applies to a limited population of patients and addresses a limited number of critical medical decisions. A POLST form complements the advance directive — it does not replace it. A POLST is a Medical Order not a legal document, that is completed by the physician POLST is for those that is Seriously ill or frail (any age), AD is for everyone POLST doesn't appoint a suraget, whereas an AD does It has Specific medical orders for treatment wishes during a medical emergency, not "general wishes" . EMS can use a POLST, they cannot use an AD A POLST is easy to locate, patient has the original and the copy is located on the medical file

How should the patient's quality of life be considered, when considering "Direct Medical Benefit?"

CPR might appear to lack potential benefit when the patient's quality of life is so poor that no meaningful survival is expected even if CPR were successful at restoring circulatory stability. However, quality of life should be used with caution in determining whether or not CPR is indicated or has the potential to provide medical benefit, for there is substantial evidence that patients with chronic conditions often rate their quality of life much higher than would healthy people. Quality of life assessments have most credibility when the patient's values, preferences, and statements inform such assessments.

When should CPR be administered?

CPR should be administered in the absence of a valid physician's order to withhold it.

Is potentially life-shortening opioid use legal?

Canadian courts have said that it can be legal for a doctor to give a patient opioids to manage pain and other forms of suffering — even enough that they know could hasten death. The Supreme Court of Canada said in the Sue Rodriguez case in 1993 that palliative treatments that could shorten life are not illegal. If a doctor eases pain by giving drugs that they know could cause your death, it is not same as a doctor giving drugs with the goal of bringing death sooner. The difference, said the court, is in the doctor's intention. The court said that legally the difference is clear. Similarly, in 2012, the Supreme Court of British Columbia wrote in Carter v. Attorney General (Canada) "potentially life-shortening symptom relief is permissible where the physician's intention is to ease pain."

Hospice care in British Columbia

Ex in Washington: most patients receive much less because they are either referred very late or have not wanted hospice. A major problem in connecting hospice care to acute medical care is that referral implies a "switch" from curative to palliative medicine-a model that does not fit comfortably in many illnesses.

Assessing requests- substantive equality and Jordan's Principle

"It is about the Aboriginal perspective; picture yourself in the community, and see it [the request] from that perspective" When considering requests, please take into account the specific needs of the child such as: Does the child have heightened needs for the service in question as a result of an historical disadvantage? Would the failure to provide the service perpetuate the disadvantage experienced by the child as a result of their race, nationality or ethnicity? Would the failure to provide the service result in the child needing to leave the home or community for an extended period? Would the failure to provide the service result in the child being placed at a significant disadvantage in terms of ability to participate in educational activities? Is the provision of support necessary to ensure access to culturally appropriate services? Is the provision of support necessary to avoid a significant interruption in the child's care? Is the provision of support necessary in maintaining family stability?, as indicated by:the risk of children being placed in carecaregivers being unable to assume caregiving responsibilities Does the individual circumstance of the child's health condition, family or community context (geographic, historical or cultural) lead to a different or greater need for services as compared to the circumstances of other children (such as extraordinary costs associated with daily living due to a remote location)? Would the requested service support the community or family's ability to serve, protect and nurture its children in a manner that strengthens the community or family's resilience, healing and self-determination? Report a problem on this page Share this page

The mental status exam (MSE) can be divided into the following major categories:

(1) General Appearance (2) Emotions (3) Thoughts (4) Cognition (5) Judgment and Insight.

Duty to report sexual misconduct of "resident" in BC

(1) If a registrant has reasonable and probable grounds to believe that another registrant has engaged in sexual misconduct, the registrant must report the circumstances in writing to the registrar of the other registrant's college. (2) Despite subsection (1), if a registrant's belief concerning sexual misconduct is based on information given in writing, or stated, by the registrant's patient, the registrant must obtain, before making the report, the consent of (a) the patient, or (b) a parent, guardian or committee of the patient, if the patient is not competent to consent to treatment.

Six Step Protocol for Breaking Bad News

1. Getting started. -The physical setting ought to be private, with both physician and patient comfortably seated. -You should ask the patient who else needs to be present, -Its helpful to start with a question like "How are you feeling right now?" 2. Finding out how much the patient knows. -"What have you already been told about your illness?" You can begin to understand what the patient has already been told ("I have lung cancer, and I need surgery"), or how much the patient understood about what's been said ("the doctor said something about a spot on my chest x-ray"), the patients level of technical sophistication ("I've got a T2N0 adenocarcinoma"), and the patient's emotional state ("I've been so worried I might have cancer that I haven't slept for a week"). 3. Finding out how much the patient wants to know. -It is useful to ask patients what level of detail you should cover "Some patients want me to cover every medical detail, but other patients want only the big picture--what would you prefer now?" This establishes that there is no right answer, and that different patients have different styles. Also this question establishes that a patient may ask for something different during the next conversation. 4. Sharing the information. Decide on the agenda before you sit down with the patient, so that you have the relevant information at hand. The topics to consider in planning an agenda are: -diagnosis -treatment -prognosis -support or coping. However, an appropriate agenda will usually focus on one or two topics. Give the information in small chunks, and be sure to stop between each chunk to ask the patient if he or she understands ("I'm going to stop for a minute to see if you have questions"). Long lectures are overwhelming and confusing. Remember to translate medical terms into English, and don't try to teach pathophysiology. 5. Responding to the patient's feelings. "Could you tell me a bit about what you are feeling?" If you don't understand the patient's reaction, you will leave a lot of unfinished business, and you will miss an opportunity to be a caring physician. Learning to identify and acknowledge a patient's reaction is something that definitely improves with experience, if you're attentive, but you can also simply ask ("Could you tell me a bit about what you are feeling?"). 6. Planning and follow-through. At this point you need to synthesize the patient's concerns and the medical issues into a concrete plan that can be carried out in the patient's system of health care. Outline a step-by-step plan, explain it to the patient, and contract about the next step. Be explicit about your next contact with the patient ("I'll see you in clinic in 2 weeks") or the fact that you won't see the patient ("I'm going to be rotating off service, so you will see Dr. Back in clinic"). Give the patient a phone number or a way to contact the relevant medical caregiver if something arises before the next planned contact.

What sorts of interventions require informed consent?

All health care interventions require some kind of consent Most health care institutions,have policies that state which health interventions require a signed consent form. These signed forms are the culmination of a dialogue required to foster the patient's informed participation in the clinical decision.

Can I request medical assistance in dying in advance of experiencing suffering or receiving a diagnosis?

According to federal law, medical assistance in dying cannot be provided based on an advance request and can only be provided after two independent assessors have determined that all of the eligibility criteria set out in the Criminal Code have been met ie: diagnosed with a grievous and irremediable medical condition and experience intolerable suffering

What is my responsibility when a patient endangers her health by refusing a treatment?

Adults have a moral and legal right to make decisions about their own health care, including the right to refuse treatments that may be life-saving. The physician has a responsibility to make sure that the patient understands the possible and probable outcomes of refusing the proposed treatment. The physician should attempt to understand the basis for the patient's refusal and address those concerns and any misperceptions the patient may have. In some cases, enlisting the aid of a leader in the patient's cultural or religious community may be helpful.

Are advance directives legally binding?

Advance directives are recognized in one form or another by legislative action in all 50 states (in Washington, see RCW 70.122). If the directive is constructed according to the outlines provided by pertinent state legislation, they can be considered legally binding. In questionable cases the medical center's attorney or ethics advisory committee can provide guidance on how to proceed

stillbirths

After 20 weeks gestation the law against secretly disposing of a child's dead body does not apply to miscarried or aborted fetuses, only to stillbirths (which are defined in Canada as after 20 weeks gestation) These decisions mean that a person's right to abortion outweighs any possible legal recognition of fetuses, including even a woman's disposal of her fetal remains. (It also means that self-induced abortion is legal in Canada.)

some questions or aids that I could use for advance care planning?

Are there any life-sustaining treatments that you know you would want to receive regardless of the circumstances, or would not want to receive under any circumstances? If so, what are they and why do you feel this way about them? For other situations in which you would not be able to communicate your preferences, such as being in a coma, what do you think should be the goals for your care? For example, should the goal of care be to prolong your life, improve or maintain your function and/or quality of life, provide comfort care, or something else? For each of the circumstances that you just identified, what do you think should be the goals for your care? For example, should the goal of care be to prolong your life, improve or maintain your function and/or quality of life, provide comfort care, or something else? Are there any circumstances that you've heard about through the news or TV where you've said to yourself, "I hope that never happens to me" or "I would never want to live like that?" If so, what are they and why do you feel this way about them? Who should speak on your behalf if you become so sick you can't speak for yourself? Should your current preferences be strictly applied to future situations or serve as a general guide to your spokesperson or family member(s)? In the event that you are dying, where do you want to receive your health care? Some people have more concerns about the way they will die or dying than death itself. Do you have any fears or concerns about this?

A 23-year-old Navajo man has injured his leg after a fall. He presents to the emergency room of the reservation hospital where he is complaining of pain. His leg appears to be broken. The man requests that you call a medicine man before doing anything further. Should you find a medicine man? Should you proceed with treatment?

As a competent adult, this patient has the right to make decisions about his medical care. You must respect his wish not to be treated until he gives you permission to do so. Calling the local medicine man will show your respect for the patient and strengthen the patient's trust in you and your abilities.

What if a patient changes their mind on an advance directive

As long as a patient remains able to participate in medical decisions, both documents are revocable. Informed decisions by patients with decision-making capacity always supersede a written directive.

Jordan River Anderson

At the age of 5, Jordan River Anderson, a First Nations child from Norway House Cree Nation in Manitoba, died in the hospital while the provincial and federal governments could not agree on who was financially responsible for his home care in a medical foster home.

Strategies for maintaining a therapeutic relationship:

Be compassionate and empathic. Keep in mind that most patients whom you find frustrating to deal with have experienced significant adversity in their lives. Acknowledge and address underlying mental health issues early in the relationship. Prioritize the patient's immediate concerns and elicit the patient's expectations of the visit and their relationship with you. Set clear expectations, ground rules, and boundaries and stick to them. Have regular visits, which helps convey confidence that the patient can deal with transient flare-ups without an emergency visit. Be aware that strong negative emotions directed at you are often misplaced. The patient may be imposing feelings and attitudes onto you that they have had toward other doctors, friends, family members in the past. This is known as transference. Acknowledge the patient's feelings and set behavioral expectations. Be aware of your own emotional reactions and attempt to remove yourself so you can objectively reflect on the situation. Involve colleagues. Vent your feelings or debrief confidentially with a trusted colleague so that your negative emotions are kept at bay during patient encounters. Recognize your own biases. For example, patients with addictions genuinely need medical care, but the behaviors associated with addiction are vexing for health care providers. These patients are often both vulnerable and manipulative. Be sure that you are attentive to their vulnerability, rather than focusing exclusively on their manipulative behaviors. Avoid being very directive with these patients. A tentative style tends to work better. Remember that you provide something many of these patients do not have-a steady relationship with someone who genuinely wants to help them. This in itself can improve the patient's health, even in the absence of medical treatment. Prepare for these visits. Keep in mind your goals of care and make a strategy for the encounter before it occurs. "What I hear from you is that . . . Did I get that right?" "How do you feel about the care you are receiving from me? It seems to me that we sometimes don't work together very well." "It's difficult for me to listen to you when you use that kind of language." "You seem quite upset. Could you help me understand what you are going through right now?" "What's your understanding of what I am recommending, and how does this fit with your ideas about how to solve your problems?" "I wish I (or a medical miracle) could solve this problem for you, but the power to make the important changes is really yours."

Who is eligible for medical assistance in dying? In order to be eligible to receive medical assistance in dying, a person must meet all of the following criteria:

Be eligible for health services publicly funded by a government in Canada, such as being registered or eligible for B.C.'s Medical Services Plan; Be at least 18 years of age and capable of making decisions about their health; Have made a voluntary request for medical assistance in dying that, in particular, was not made as a result of external pressure; Have given informed consent to receive medical assistance in dying after being informed of the means that are available to relieve their suffering, including palliative care; and Have a grievous and irremediable medical condition, which means:they have a serious and incurable illness, disease or disability*;they are in an advanced state of decline that cannot be reversed; andthat illness, disease or disability or that state of decline causes them enduring physical or psychological suffering that is intolerable to them and cannot be relieved under conditions that they consider acceptable. * Note: For the purposes of medical assistance in dying eligibility, a mental illness is not considered to be an "illness, disease, or disability." This restriction will be repealed two years after the new legislation comes into force.

A young accident victim has been in a persistent vegetative state for several months and family members have insisted that "everything possible" be done to keep the patient alive. What are your professional obligations?

Case 1 illustrates the possible conflicts that can arise with patients or family members about withholding or withdrawing futile interventions. If you and other members of the health care team agree that the interventions in question would be futile, the goals of treatment should be clarified. It can be helpful to ask patients and family members to also articulate their goals, which may reveal some agreement among parties. Once goals are clear, physicians can discuss how various interventions help or frustrate these goals. This requires a process of working with the patient and/or family, and possibly drawing on other resources, such as social workers, palliative care services, hospital chaplains, and ethics consultants. If there is no professional consensus about the futility of a particular intervention, then there is no ethical basis for overriding the requests of patients and/or family members for that intervention.

What about obtaining court orders to force pregnant women to comply?

Court orders force pregnant women to forfeit their autonomy in ways not required of competent men or nonpregnant women. There is an inconsistency in allowing competent adults to refuse therapy in all cases but pregnancy. Hospital administrators, lawyers and judges have little warning of impending conflicts and little time for deliberation; this time pressure makes it unlikely that pregnant women will have adequate legal representation. Such cases call for an interdisciplinary approach, strong efforts at effective medical communication, and resources for the patient and the health care team. The most suitable ethical framework for addressing a pregnant woman's refusal of recommended care is one that recognizes the interconnectedness of the pregnant woman and her fetus but maintains as a central component respect for the pregnant woman's autonomous decision making. This approach does not restrict the obstetrician-gynecologist from providing medical advice based on fetal well-being, but it preserves the woman's autonomy and decision-making capacity surrounding her pregnancy. Pregnancy does not lessen or limit the requirement to obtain informed consent or to honor a pregnant woman's refusal of recommended treatment. The use of coercion is not only ethically impermissible but also medically inadvisable because of the realities of prognostic uncertainty and the limitations of medical knowledge. As such, it is never acceptable for obstetrician-gynecologists to attempt to influence patients toward a clinical decision using coercion. Obstetrician-gynecologists are discouraged in the strongest possible terms from the use of duress, manipulation, coercion, physical force, or threats, including threats to involve the courts or child protective services, to motivate women toward a specific clinical decision.

What are the ethical obligations of members of the interdisciplinary team in patient care?

Ethically, every member of the operating room team has separate obligations, or duties, toward patients, which are based on the provider's profession, scope of practice and individual skills. Team members also have ethical obligations to treat each other in a respectful and professional manner. Relationships between professionals on the multidisciplinary team are by their nature unequal ones. Different knowledge and experience in specific issues both ethically and legally imparts unequal responsibility and authority to those care providers with the most knowledge and experience to handle them. But also because of differences in training and experience, each member of the team brings different strengths. Team members need to work together in order to best utilize the expertise and insights of each member.

What if the family disagrees with the DNAR order?

Ethicists and physicians are divided over how to proceed if the family disagrees with the recommendation to forgo attempting CPR. If there is disagreement, every reasonable effort should be made to clarify questions and communicate the risks and potential benefits of CPR with the patient or family. In many cases, this conversation will lead to resolution of the conflict. However, in difficult cases, an ethics consultation can prove helpful.

DNR in the OR- emergencies

Even in emergencies, physicians have an ethical obligation to recognize and respect patient autonomy. Whenever possible, physicians should obtain input from the patient, or when the patient is incapacitated, from appropriate surrogates, regarding the status of the patient's DNR orders in the OR. In the absence of such input, consensus should be reached among the caregivers about the medical benefits or futility of CPR. In any case, medical care of the patient in the absence of patient input should be directed toward realizing, to the best of the physician's ability and knowledge, the patient's goals.

3. If you have the choice of giving a transplant to a successful elderly member of the community and a 20-year-old drug addict - how do you choose?

In this situation, I am faced with an ethical dilemma of giving an organ transplant to a successful elderly member of the community or to a 20-year-old drug addict. First, I must remain non-judgmental and gather information first. I must not jump to conclusions about age or the fact that one patient appears to have a dependence on drugs. First, I will ensure that both patients are indeed matches to receive this transplant. I need to find out about both patients' conditions and what led them to requiring an organ transplant. I must also find out about the prognoses of both patients post-transplant. -Which patient is more likely to recover and gain the most from this transplant, including not just quantity but quality of life? Since organs are a very limited resource, I need more information first. I also should not feel that only my opinion is important here. Healthcare is a team-based environment, and I should seek out others' thoughts, including the patients' physicians, nurses, other healthcare staff, and the hospital ethics or transplant board, which is often involved in finalizing these decisions. There are 2 outcomes here: either the elderly patient or the young patient. In either case, the decision will be made non-judgmentally and based on which patient will be able to have a strong recovery from this transplant. For the patient who does not receive the transplant, the healthcare team must discuss this with the patient and their loved ones, and ensure they understand that not receiving the transplant right now does not mean that we are going to stop treatment or looking for ways to improve their condition, and that they are still on the transplant list. Moving forward, I will ensure our hospital has a clear set of guidelines in place so organs are allocated on a non-judgmental basis with clear policies in place. To summarize, I would gather information and ensure we have been non-judgmental in deciding which patient to provide the transplant to, and ensure guidelines are in place for future situations like this one. Thank you and I look forward to addressing any further questions you have.

First Nations Child and Family Services

Indigenous Services Canada's (ISC) First Nations Child and Family Services program funds prevention and protection services to support the safety and well-being of First Nations children and families living on reserve. January 4, 2022, Canada announced that it had reached an Agreement-in-Principle with the Assembly of First Nations, the First Nations Child and Family Caring Society, the Chiefs of Ontario and the Anishnawbe Aski Nation on the long-term reform of the First Nations Child and Family Services Program to: better support First Nations and First Nations child and family services agencies in providing culturally-based and substantially equal family supports reduce the number of Indigenous children in care and keep children with their families, where they belong ISC provides funding to First Nations child and family services agencies, which are established, managed and controlled by First Nations and delegated by provincial authorities to provide prevention and protection services uses a prevention-based funding model to support early intervention and alternatives to traditional institutional care and foster care, such as the placement of children with family members in a community setting. The program provides 3 streams of funding: Operations: core and operational funding for protection services (such as salaries and overhead) Prevention: resources for enhanced prevention services Maintenance: direct costs of placing First Nations children into temporary or permanent care out of the parental home (such as foster care rates and group home rates)

What ethical issues are associated with research in CAM?

Informed consent: The regulations stipulate that the informed consent document and process must accurately describe reasonably anticipated risks and potential benefits. Yet in many instances CAM therapies have not had systematic safety or efficacy data collected, and are ratified by historical anecdotal evidence. The informed consent document should state that although a therapy might have been in widespread use, rigorous safety data may not be available. Misconceptions: A prospective research subject must weigh for themselves the balance of risks and benefits that will accrue from their participation. The therapeutic misconception is the (documented) belief held by many research subjects that they will benefit from participating in a research study (allopathic or CAM), irrespective of disclaiming language in the informed consent form. This misconception weighs in favor of participation. Additionally, there is a widely held notion that if something is "natural" it must be safe, or beneficial. This misconception also weighs in favor of participation by reducing the perceived risks associated with the study. In CAM research it is imperative to impress upon potential subjects that the risks and benefits of participation are more difficult to anticipate than they are for better-studied interventions. Study design: The Belmont principle of beneficence dictates that there be a reasonable likelihood of obtaining useful data from a study. As discussed above, the difficulties inherent in generating sound designs in CAM research challenge adherence to this principle. Research study designs must undergo rigorous review by scientists and clinicians well versed in the CAM modality being tested.

Jose is a 62-year-old man who just had a needle biopsy of the pancreas showing adenocarcinoma. You run into his brother in the hall, and he begs you not to tell Jose because the knowledge would kill him even faster. A family conference to discuss the prognosis is already scheduled for later that afternoon. How should you handle this?

It is common for family members to want to protect their loved ones from bad news, but this is not always what the patient himself would want. It would be reasonable to tell Jose's brother that withholding information can be very bad because it creates a climate of dishonesty between the patient and family and medical caregivers; also, that the only way for Jose to have a voice in the decision making is for him to understand the medical situation. Ask Jose how he wants to handle the information in front of the rest of the family, and allow for some family discussion time for this matter. In some cultures it is considered dangerous to talk about prognoses and to name illnesses (e.g., the Navajo). If you suspect a cultural issue it is better to find someone who knows how to handle the issue in a culturally sensitive way than to assume that you should simply refrain from providing medical information. For many invasive medical interventions which require a patient to critically weigh burdens and benefits, a patient will need to have some direct knowledge of their disease in Western terms in order to consider treatment options.

An elderly patient with irreversible respiratory disease is in the intensive care unit where repeated efforts to wean him from ventilator support have been unsuccessful. There is general agreement among the health care team that he could not survive outside of an intensive care setting. The patient has requested antibiotics should he develop an infection and CPR if he has a cardiac arrest. Should a distinction be made between the interventions requested by the patient? Should the patient's age be a factor?

Like Case 1, Case 2 illustrates possible conflicts that can arise with patients or family members about withholding or withdrawing futile interventions. Ventilator support clearly produces a physiologic effect by contracting and expanding the patient's lungs with oxygen. Yet this does not suffice to show that the treatment helps the patient, which is medicine's goal. If the patient will never leave the intensive care unit, does ventilator support constitute a benefit to the patient? A central issue this case raises concerns qualitative futility, namely: does the patient's quality of life fall well below a threshold considered minimal? If you and other members of the health care team agree that the interventions in question would be qualitatively futile, the goal should be to withdraw or withhold these interventions. Achieving this goal requires working in tandem with the patient and/or family, as well as drawing upon resources, such as social workers, hospital chaplains, and ethics committees. If there is no professional consensus about the futility of a particular intervention, then there is no ethical basis for overriding the requests of patients and/or family members for that intervention.

What are the limitations of living wills?

Living wills cannot cover all conceivable end-of-life decisions. There is too much variability in clinical decision making to make an all-encompassing living will possible. Persons who have written or are considering writing advance directives should be made aware of the fact that these documents are insufficient to ensure that all decisions regarding care at the end of life will be made in accordance with their written wishes. Moreover, the language that is often employed in advance directives is frequently imprecise. What seems clear to the author may seem cloudy to others when reviewed in a clinical situation. A partial remedy to this limitation is to strongly encourage patients to communicate preferences and values to both their medical providers and family/surrogate decision makers, and to encourage them to identify a health care agent (through a durable power of attorney for health care) where appropriate. Another potential limitation of advance directives is possible changes in the patient's preferences over time or circumstance. People often accommodate to disabilities and an old living will may become inconsistent with the patient's revised views about quality of life or other outcomes. This is yet another reason to recommend ongoing communication between patients and their physicians and family members.

How should I interpret a patient's advance directive, with ambiguous terms like "extraordinary means" and "unnaturally prolonging my life"

Living wills generally are written in ambiguous terms and demand interpretation by providers. Terms like "extraordinary means" and "unnaturally prolonging my life" need to be placed in context of the present patient's values in order to be meaningfully understood. The health care agent or a close family member often can help the care team reach an understanding about what the patient would have wanted. Of course, physician-patient dialogue is the best guide for developing a personalized advance directive.

Do different standards apply to withholding and withdrawing care?

Many clinicians feel that it is easier to not start (withhold) a treatment, such as mechanical ventilation, than to stop (withdraw) it. While there is a natural tendency to believe this, there is no ethical distinction between withholding and withdrawing treatment. In numerous legal cases, courts have found that it is equally justifiable to withdraw as to withhold life-sustaining treatments. Also, most bioethicists, including the President's Commission, are of the same opinion.

What if the patient's family asks me to withhold the truth from the patient?

Often families will ask the physician to withhold a terminal or serious diagnosis or prognosis from the patient. Usually, the family's motive is laudable; they want to spare their loved one the potentially painful experience of hearing difficult or painful facts. These fears are usually unfounded, and a thoughtful discussion with family members, for instance reassuring them that disclosure will be done sensitively, will help allay these concerns. In unusual situations, family members may reveal something about the patient that causes the physician to worry that truthful disclosure may create real and predictable harm, in which case withholding may be appropriate. These occasions, however, are rare.

Next steps Canada and the parties to the agreement-in-principle will continue to work together to reach a final settlement agreement on long-term reform.

Once a final settlement is reached and the necessary CHRT orders are made, measures will be implemented to: better meet the needs of First Nations children, youth, and families prevent Canada's discriminatory underfunding and narrow application of Jordan's Principle from recurring

How do we Define Direct Medical Benefit?

One approach to defining benefit examines the probability of an intervention leading to a desirable outcome. Outcomes following CPR have been evaluated in a wide variety of clinical situations. In general, survival rates in adults following in-hospital cardiac arrest range from 8-39% with favorable neurological outcomes in 7-14% of survivors (Meaney et al., 2010). In children, the survival rate following in-hospital cardiac arrest is closer to 27% with a favorable neurological outcome in up to one third of survivors (AHA, 2010). In general, these statistics represent the population as a whole and do not necessarily reflect the chance of survival for an individual patient. Hence, multiple factors, including both the distal and proximal causes for cardiopulmonary arrest, must be considered to determine whether or not CPR has the potential to promote survival

What should I do if a patient asks me for physician aid-in-dying (PAD)?

One of the most important aspects of responding to a request for PAD is to be respectful and caring. Virtually every request represents a profound event for the patient, who may have agonized over his situation. The patient's request should be explored, to better understand its origin and to determine if there are other interventions that may help ameliorate the concerns that motivated the request. In most cases, there are alternatives in palliative and hospice care that likely will address most of the patient's concerns.

Is professionalism compatible with the restrictions sometimes placed on physician's judgments in managed care?

One of the principal attributes of professionalism is independent judgment about technical matters relevant to the expertise of the profession. The purpose of this independent judgment is to assure that general technical knowledge is appropriately applied to particular cases. Today, many physicians work in managed care situations that require them to abide by policies and rules regarding forms of treatment, time spent with patients, use of pharmaceuticals, etc. In principle, such restrictions should be designed to enhance and improve professional judgment, not limit it. For example, requiring consultation is ethically obligatory in doubtful clinical situations; penalizing consultation for financial reasons would be ethically wrong. Also, requiring physicians to adhere to practice guidelines and to consult outcome studies may improve professional judgment; requiring blind adherence to those guidelines may be a barrier to the exercise of professional judgment. The presence of rules, policies and guidelines in managed care settings requires the physicians who work in these settings to make such judgments and to express their reasoned criticism of any that force the physician to violate the principles of professionalism.

Can parents refuse to provide their children with necessary medical treatment on the basis of their beliefs?

Parents have legal and moral authority to make health care decisions for their children, as long as those decisions do not pose a significant risk of serious harm to the child's health. Parents should not be permitted to deny their children medical care when that medical care is likely to prevent substantial harm or suffering. If necessary, the physician may need to pursue a court order or seek the involvement of child protective services in order to provide treatment against the wishes of the parents. Nevertheless, the physician must always take care to show respect for the family's beliefs and a willingness to discuss reasonable alternatives with the family.

What if maternal decisions seem to be based on unusual beliefs?

Parents are granted wide discretion in making decisions about their children's lives. However, when the exercise of certain beliefs would disadvantage the child's health in a serious way, there are limits in exercising this discretion see the pediatric portion of the Cross-Cultural Issuestopic page).

What kinds of treatment can parents choose not to provide to their children?

Parents have the right to refuse medical treatments when doing so does not place the child at significant risk of substantial harm or suffering. For example, parents have the right to refuse routine immunizations for their children on religious or cultural grounds.

What is Canada's obligation under Jordan's Principle with respect to substantive equality?

Pursuant to the CHRT May 26, 2017 decision as amended, the Government of Canada is to ensure substantive equality in the provision of services to the child, to ensure culturally appropriate services and to safeguard the best interests of the child. This requires Canada to provide all First Nations children, on and off reserve, and Indigenous children ordinarily living on reserve, with publicly funded benefits, supports, programs, goods and services in a manner and according to a standard that meets their particular needs and circumstances, on a substantively equal basis with non-First Nations children.

two types of "Medical futility"

Quantitative futility, where the likelihood that an intervention will benefit the patient is exceedingly poor, and Qualitative futility, where the quality of benefit an intervention will produce is exceedingly poor. Both quantitative and qualitative futility refer to the prospect that a specific treatment will benefit (not simply have a physiological effect) on the patient.

IC: How do you know when you have provided enough information about a proposed intervention?

Reasonable patient standard: what would the average patient need to know in order to be an informed participant in the decision? This standard focuses on considering what a typical patient would need to know in order to understand the decision at hand. The best approach to the question of how much information is enough is one that meets both your professional obligation to provide the best care and respects the patient as a person, with the right to a voice in health care decisions.

Walk in clinic vs family doctor

Recognize the interplay of time a space between your body and time and space -family doctors are able to identify the patterns in the individuals, levels of symptoms, and the population in general -its a speciality of relationship Walk in: Time and place no access to records

What are the arguments in favor of physician aid-in-dying (PAD)?

Respect for autonomy: Decisions about time and circumstances of death are personal. Competent people should have right to choose the timing and manner of death. Justice: Justice requires that we "treat like cases alike." Competent, terminally ill patients have the legal right to refuse treatment that will prolong their deaths. For patients who are suffering but who are not dependent on life support, such as respirators or dialysis, refusing treatment will not suffice to hasten death. Thus, to treat these patients equitably, we should allow assisted death as it is their only option to hasten death. Compassion: Suffering means more than pain; there are other physical, existential, social and psychological burdens such as the loss of independence, loss of sense of self, and functional capacities that some patients feel jeopardize their dignity. It is not always possible to relieve suffering. Thus PAD may be a compassionate response to unremitting suffering. Individual liberty vs. state interest: Though society has strong interest in preserving life, that interest lessens when a person is terminally ill and has strong desire to end life. A complete prohibition against PAD excessively limits personal liberty. Therefore PAD should be allowed in certain cases. Honesty & transparency: Some acknowledge that assisted death already occurs, albeit in secret. The fact that PAD is illegal in most states prevents open discussion between patients and physicians and in public discourse. Legalization of PAD would promote open discussion and may promote better end-of-life care as patients and physicians could more directly address concerns and options

UBC Guiding Principles

Respect the dignity, individuality and needs of our patients. Strive to discover new knowledge that will advance state-of-the-art care. Ensure effective and innovative teaching of our students. Optimize our organization to promote both professional and personal fulfillment. Value the efforts and achievements of teams and individuals who work to face the challenges of modern medicine. These activities will ensure the respect of our peer organizations and recognition as a world-class Department of Medicine.

Why is it important to respect what appear to me to be idiosyncratic beliefs?

Respecting the beliefs and values of your patient is an important part of establishing an effective therapeutic relationship. Failure to take those beliefs seriously can undermine the patient's ability to trust you as her physician. It may also encourage persons with non-mainstream cultural or religious beliefs to avoid seeking medical care when they need it.

What safeguards are in place for medical assistance in dying?

Safeguards - Natural Death Is Reasonably Foreseeable Safeguards - Natural Death Is Not Reasonably Foreseeable (NEW) Provincial Safeguards

What are the arguments against physician aid-in-dying (PAD)?

Sanctity of life: Religious and secular traditions upholding the sanctity of human life have historically prohibited suicide or assistance in dying. PAD is morally wrong because it is viewed as diminishing the sanctity of life. Passive vs. Active distinction: There is an important difference between passively "letting die" and actively "killing." Treatment refusal or withholding treatment equates to letting die (passive) and is justifiable, whereas PAD equates to killing (active) and is not justifiable. Potential for abuse: Vulnerable populations, lacking access to quality care and support, may be pushed into assisted death. Furthermore, assisted death may become a cost-containment strategy. Burdened family members and health care providers may encourage loved ones to opt for assisted death and the protections in legislation can never catch all instances of such coercion or exploitation. To protect against these abuses, PAD should remain illegal. Professional integrity: Historical ethical traditions in medicine are strongly opposed to taking life. For instance, the Hippocratic oath states, "I will not administer poison to anyone where asked," and I will "be of benefit, or at least do no harm." Furthermore, some major professional groups such as the American Medical Association and the American Geriatrics Society oppose assisted death. The overall concern is that linking PAD to the practice of medicine could harm both the integrity and the public's image of the profession. Fallibility of the profession: The concern here is that physicians will make mistakes. For instance there may be uncertainty in diagnosis and prognosis. There may be errors in diagnosis and treatment of depression, or inadequate treatment of pain. Thus the State has an obligation to protect lives from these inevitable mistakes and to improve the quality of pain and symptom management at the end of life.

The key values identified in the Touchstones of Hope, as outlined below, are to be respected to achieve substantive equality in the provision of services, products and supports, under Jordan's Principle:

Self-determination First Nations peoples are in the best position to make decisions that affect First Nations children, youth, families and communities. First Nations peoples must meaningfully participate in the development and implementation of Jordan's Principle on a regular and ongoing basis. Culture and language Culture and language are the foundations of health and well-being for First Nations peoples. Jordan's Principle recognizes this and requires that approved products, services and supports are culturally appropriate. Holistic approach The holistic needs of a child must be met. These needs will be informed by historical and cultural factors, such as residential schools, intergenerational trauma, colonization, racism and intersectional discrimination. Products, services and supports must meet the needs of the child in the context of their family and community and be child-centred, focused on promoting the health and well-being of the child's mind, body, spirit and emotions. Structural interventions Jordan's Principle requires the eliminating of systemic barriers that have resulted from racism and colonialism by challenging the existing systems to fully meet the needs of First Nations children. Non-discrimination Non-discrimination underlies Jordan's Principle by ensuring that First Nations children receive the products, services and supports they need regardless of where they live. It challenges historical practices and structural barriers and strives for equal access to health, social and educational systems in order to achieve equal outcomes.

SPIKES (breaking bad news)

Setting Perception Invitation Knowledge Empathy Summary

What should be included in a discussion of DNR orders in the OR with the patient or patient's surrogates?

Since the goal of medical therapy is to provide meaningfulbenefits to the patient, discussion of DNR orders in the OR should center around the patient's goals for surgical therapy. Patients may have fears of "ending up a vegetable" on a ventilator after surgery, for example. In those cases, discussion should center around the positive prognosis for patients who have CPR in the OR, together with reassurance that the patient's stated wishes in their advanced directive regarding ventilatory support would be followed postoperatively after anesthetic effects are ruled out as a cause of ventilatory depression. Most authorities now agree that a "smorgasbord" or checklist "yes-or-no" approach to the various procedures in the operating room is confusing and counterproductive to the purpose of DNR discussions. require documentation of discussion of DNR orders with the patient or appropriate surrogates, anesthesiologists and surgeons should nevertheless approach the patient about to undergo surgery with sensitivity to the fact that they may be unaware of their DNR order. If this proves to be the case, a full discussion of the DNR order should be undertaken prior to proceeding.

What about "slow codes" or "show codes"?

Slow codes and show codes are forms of "symbolic resuscitation." A "slow code" is an act performed by the health care providers that resembles CPR yet is not the full effort of resuscitation while a "show code" is a short and vigorous resuscitation performed to benefit the family while minimizing harm to the patient Slow and show codes are ethically problematic. In general, performing slow and show codes undermines the rights of patients to be involved in clinical decisions, is deceptive, and violates the trust that patients have in health care providers.

General -What is substantive equality?

Substantive equality is a legal principle that refers to the achievement of true equality in outcomes. It is achieved through equal access, equal opportunity and, most importantly, the provision of services and benefits in a manner and according to standards that meet any unique needs and circumstances, such as cultural, social, economic and historical disadvantage. Substantive equality is both a process and an end goal relating to outcomes that seeks to acknowledge and overcome the barriers that have led to the inequality in the first place. When substantive equality in outcomes does not exist, inequality remains. Achieving substantive equality for members of a specific group requires the implementation of measures that consider and are tailored to respond to the unique causes of their historical disadvantage as well as their historical, geographical and cultural needs and circumstances. First Nations children have experienced historical disadvantage due to Canada's repeated failure to take into account their best interest as well as their historical, geographical and cultural needs and circumstances. For this reason, substantive equality for First Nations children will require that government policies, practices and procedures impacting them take account of their historical, geographical and cultural needs and circumstances and aim to safeguard the best interest of the child as articulated in the United Nations Committee on the Rights of the Child General Comment 11.

Understanding substantive equality

Substantive equality is the recognition that not all people start off from the same position and that these unequal opportunities make it more difficult for some to be successful. Treating everyone the same is only fair if they are starting from the same position. Substantive equality seeks to address the inequalities that stem from an individual's particular circumstances, to help put them at the same position and give them the same opportunities as others.

UBC "Our Mission"

The University of British Columbia's MD Undergraduate Program strives to deliver exemplary distributed medical education that prepares future physicians to collaborate with patients and their circle of support in providing culturally safe, high-quality healthcare for the diverse and changing populations in BC and beyond, including Indigenous Peoples, people living in rural and remote communities, and those who disproportionately experience adverse outcomes in health care*. Our graduates are prepared to engage with diverse communities and to embrace generalist or specialist postgraduate training to further prepare them for medical practice and to contribute innovative improvements to healthcare in BC and globally. The Program is grounded in the Faculty of Medicine's vision "Transforming Health for Everyone," and embodies the values of respect, integrity, compassion, collaboration and equity. *People who disproportionately experience adverse outcomes in healthcare include but are not limited to those experiencing inequity due to their race, ethnicity, religion, spiritual beliefs, gender identity, gender expression, sexuality, socioeconomic status, or other markers of human diversity; older adults living with complex health conditions; newcomers to Canada, people whose first language is not English; people living with disabilities and/or chronic health needs, including people living with mental illness and people who use substances.

Who is the decision-maker regarding the nature of medical care administered to a newborn infant?

The biological parents (or parent) have authority regarding the decisions for their child (known as parental authority). This is a universally granted right regardless of the parents' age or other contextual features, unless the parents are declared not competent or otherwise unfit to serve as the child's proxy. There are some limits to such authority

What if the truth could be harmful?

There are many physicians who worry about the harmful effects of disclosing too much information to patients. Assuming that such disclosure is done with appropriate sensitivity and tact, there is little empirical evidence to support such a fear. If the physician has some compelling reason to think that disclosure would create a real and predictable harmful effect on the patient, it may be justified to withhold truthful information.

Determining whether a child is capable-Confidentiality

There is no set age when a child becomes capable. Doctors have to use their best judgment in each case to decide if a child is capable. Courts are flexible in deciding if a child is capable. It depends on how mature the child is and how serious the medical treatment is. A very young child may be able to consent to the dressing of a wound. The law considers a child capable if they understand the need for the health care, what the care involves, and the consequences (the benefits and risks) of getting the care — or not getting the care. If a health care provider explains these things to the child and is satisfied the child understands them, and that the health care is in the child's best interests, they can treat the child if the child consents to the care. The provider does not need the consent of the child's parents or guardians. The child might have to sign a consent form. Generally, if a child is capable of consenting to health care, they are also capable of making a decision to refuse health care.

What are the key procedural safeguards for MAiD?

Track One For a person on Track One, their medical or nurse practitioner must: be of the opinion that they meet the eligibility criteria set out above ensure that the person's request for medical assistance in dying was signed and dated by the person - or if the person requesting medical assistance in dying is unable to sign and date the request, another person — who is at least 18 years of age, who understands the nature of the request for medical assistance in dying and who does not know or believe that they are a beneficiary under the will of the person making the request, or a recipient, in any other way, of a financial or other material benefit resulting from that person's death — may do so in the person's presence, on the person's behalf and under the person's express direction ensure the person's request for MAiD was signed and dated after the person was informed by a medical practitioner or nurse practitioner that the person has a grievous and irremediable medical condition ensure that the person has been informed that they may, at any time and in any manner, withdraw their request* ensure that another medical practitioner or nurse practitioner has provided a written opinion confirming that the person meets all of the eligibility criteria be satisfied that they and the other medical practitioner or nurse practitioner conducting assessments are independent if the person has difficulty communicating, take all necessary measures to provide a reliable means by which the person may understand the information that is provided to them and communicate their decision; and immediately before providing the medical assistance in dying, give the person an opportunity to withdraw their request and ensure that the person gives express consent to receive medical assistance in dying* There is no minimum waiting period between the day the request was signed and the day MAiD is provided. Track Two If a person's natural death has not become reasonably foreseeable, then they proceed along "Track Two" with respect to additional procedural safeguards. For a person on Track Two, their medical or nurse practitioner must: (a) be of the opinion that the person meets all of the eligibility criteria set out above (b) ensure that the person's request for medical assistance in dying was made in writing and signed and dated by the person- or if the person requesting medical assistance in dying is unable to sign and date the request, another person — who is at least 18 years of age, who understands the nature of the request for medical assistance in dying and who does not know or believe that they are a beneficiary under the will of the person making the request, or a recipient, in any other way, of a financial or other material benefit resulting from that person's death — may do so in the person's presence, on the person's behalf and under the person's express direction (c) be satisfied that the person's request for MAiD was signed and dated after the person was informed by a medical practitioner or nurse practitioner that the person has a grievous and irremediable medical condition (d) ensure that the person has been informed that the person may, at any time and in any manner, withdraw their request (e) ensure that another medical practitioner or nurse practitioner has provided a written opinion confirming that the person meets all of the eligibility criteria set out above; (e.1) if neither they nor the other medical practitioner or nurse practitioner referred to in paragraph (e) has expertise in the condition that is causing the person's suffering, ensure that they or the medical practitioner or nurse practitioner referred to in paragraph (e) consult with a medical practitioner or nurse practitioner who has that expertise and share the results of that consultation with the other practitioner (f) be satisfied that they and the medical practitioner or nurse practitioner referred to in paragraph (e) are independent (g) ensure that the person has been informed of the means available to relieve their suffering, including, where appropriate, counselling services, mental health and disability support services, community services and palliative care and has been offered consultations with relevant professionals who provide those services or that care (h) ensure that they and the medical practitioner or nurse practitioner referred to in paragraph (e) have discussed with the person the reasonable and available means to relieve the person's suffering and they and the medical practitioner or nurse practitioner referred to in paragraph (e) agree with the person that the person has given serious consideration to those means (i) ensure that there are at least 90 clear days between the day on which the first assessment under this subsection of whether the person meets the eligibility criteria set out above begins and the day on which medical assistance in dying is provided to them or — if the assessments have been completed and they and the medical practitioner or nurse practitioner referred to in paragraph (e) are both of the opinion that the loss of the person's capacity to provide consent to receive medical assistance in dying is imminent — any shorter period that the first medical practitioner or nurse practitioner considers appropriate in the circumstances (j) if the person has difficulty communicating, take all necessary measures to provide a reliable means by which the person may understand the information that is provided to them and communicate their decision and (k) immediately before providing the medical assistance in dying, give the person an opportunity to withdraw their request and ensure that the person gives express consent to receive medical assistance in dying.

Can I make allocation decisions based on judgments about "quality of life"?

Under conditions of scarcity, the question may arise whether a patient's quality of life seems so poor that use of extensive medical intervention appears unwarranted. When this question is raised, it is important to ask a few questions. First, who is making this quality of life judgment, the care team, the patient, or the patient's family? Several studies have shown that physicians often rate the patient's quality of life much lower than the patient himself does. If the patient is able to communicate, you should engage her in a discussion about her own assessment of her condition. When considering quality of life, you should also ask: What criteria are being used to make the judgment that the quality of life is unacceptable? These criteria are often unspoken and can be influenced by bias or prejudice. A dialogue between care givers and the patient can surface some underlying concerns that may be addressed in other ways. For example, residents on a medical floor in an urban public hospital may get discouraged with the return visits of a few chronically ill alcoholic patients and suggest that money is being wasted that could be used for prenatal care or other medically beneficial projects. While the residents' frustration is understandable, it would be helpful to consider other ways they might interrupt this vicious cycle of repeat admissions. How could this patient population be supported in ways that might improve health? Quality of life judgments based on prejudices against age, ethnicity, mental status, socioeconomic status, or sexual orientation generally are not relevant to considerations of diagnosis and treatment. Furthermore, they should not be used, explicitly or implicitly, as the basis for rationing medical services.

When looking for the capability for your patient to consent, you should assess the patient's ability to:

Understand his or her situation, Understand the risks associated with the decision at hand, and Communicate a decision based on that understanding. When this is unclear, a psychiatric consultation can be helpful. Of course, just because a patient refuses a treatment does not in itself mean the patient is incompetent. Competent patients have the right to refuse treatment, even those treatments that may be life-saving. Treatment refusal may, however, be an indication that it is necessary to pause to discuss further the patient's beliefs and understanding about the decision, as well as your own.

What about the patient whose decision making capacity varies from day to day?

You should do what you can to catch a patient in a lucid state - even lightening up on the medications if necessary and safe - in order to include her in the decision making process. if a careful assessment is done and documented at each contact, and during lucid periods the patient consistently and persistently makes the same decision over time, this may constitute adequate decisional capacity for the question at hand.

How should I advise a patient if she doesn't have anyone to name as a proxy?

You should inform the patient that the best course of action under these circumstances is to write down her wishes and give a copy to her health care providers. She should fill out a legal form, such as a living will, with as much detail as possible, and then include more detailed values and preferences to provide a better understanding of her wishes. Copies of completed documents should be provided to every health care clinic/facility where the patient receives care

Is VSED as an alternative to MAiD legal in Canada?

VSED is a legal alternative to MAiD for most people in Canada. Competent patients can legally cause their own death through VSED because they can legally refuse artificial and oral nutrition and hydration. The Canadian common law clearly holds that competent adult patients can refuse medical treatment even when that refusal has the direct consequence of death. Likewise, previously competent adult patients may refuse medical treatment through prior expressed wishes where the consequence of that refusal is death. And given the principles of bodily integrity and autonomy that are entrenched in the common law, it can be persuasively argued that competent adult patients have a common law right to refuse oral and artificial nutrition and hydration.

Palliative care physicians recommend the following process for evaluating and responding to requests to MAID:

Wait to directly respond to the request until you have explored the reasons for the patient's request. Discuss various ways of addressing the patient's pain, suffering, hopes, and fears. If time permits, tell the patient that you would like to talk more about this at a subsequent appointment. That gives both you and the patient time to prepare for a fuller exploration of PAD as well as other palliative treatments, hospice, etc. Evaluate for depression or other psychiatric conditions and treat appropriately. Assess the patient's decision-making competence. Engage in discussion surrounding the patient's diagnosis, prognosis, and goals for care. Make sure to assess patient understanding. Evaluate patient's physical, mental, social, and spiritual suffering. Be sure to take into account the patient's support system as well as personal and professional pressures and stressors. Discuss all alternative options, such as palliative care and hospice. Consult with professional colleagues regarding the patient's situation. Where appropriate, ask for help from a palliative care specialist to assure that all options have been explored. Help the patient complete advance directives, DNR orders and POLST forms, as appropriate and ensure that preferences are followed.

How to raise advance care planning without scaring patients?

You can raise advance care planning as one of many health promotion activities. These discussions are aimed at avoiding harms (over- and under-treatment), and promoting benefits (treatments tailored to the patient's goals). You should reassure the patient that raising this issue does not mean that there is something unspoken to worry about. You also may tell the patient that this topic is difficult for many patients and that you will understand if she does not want to come to any conclusions during this discussion. You may want to refer the patient to one of several reputable internet based advance care planning sites to stimulate further thought and preliminary steps in advance care planning

What if the patient or family requests an intervention that the health care team considers futile?

You have a duty as a physician to communicate openly with the patient or family members about interventions that are being withheld or withdrawn and to explain the rationale for such decisions. The aim of respectful communication should be to elicit the patient's goals, explain the goals of treatment, and help patients and families understand how particular medical interventions would help or hinder their goals and the goals of treatment. It is important to approach such conversations with compassion. For example, rather than saying to a patient or family, "there is nothing I can do for you," it is important to emphasize that "everything possible will be done to ensure the patient's comfort and dignity."

Under what circumstances should I call the ethics consultant/service? You should consider asking for a case consultation when two conditions are met:

You perceive that there is an ethical problem in the care of patients, and Health care providers have not been able to establish a resolution that is agreed upon by the patient/surrogate and the clinicians caring for the patient

To access MAiD in Canada* you must:

be eligible for health care services in Canada (or you would be eligible but for a minimum period of living in a province or territory or a waiting period for eligibility) be at least 18 years old be capable of making decisions with respect to your health have asked for MAiD yourself and no one is pressuring you to ask for it have been told about other available ways to relieve your suffering, including palliative care have given informed consent to receive MAiD have what is called a "grievous and irremediable medical condition"

Respectful behavior begins with

both listening to and considering the input of other professionals. Ask yourself whether your perception of whether you are respected depends more upon whether the other party agrees with you, or whether, despite disagreeing, they listened and acknowledged your point of view. it is expected, that members of the patient care team will disagree at times, it is never acceptable for disagreements to be verbalized in an unprofessional manner. "I acknowledge and respect your perspective in this matter, but for the following reasons. I disagree with your conclusions, and believe I should do something else..."

a physician suggest that the physician's role (as a generalist in spirituality) is to:

briefly screen patient's spiritual needs as they relate to health care and to refer to the chaplain (a specialist in pastoral care), as appropriate


Conjuntos de estudio relacionados

Macro Midterm #5 (Ch 15,16,17, & 18) NOTES

View Set

ISTQB Foundation Extension Agile Tester Chapter 3: Agile Testing Methods, Techniques, and Tools

View Set

(Chapter 16 HW) Exchange Rates and International Capital Flows

View Set

The Great Gatsby Vocabulary Chapters 1-3

View Set

Chapter 48: Care of Patient with Ear Problems

View Set